Test Bank – ACP: Rheumatology

Introduction to the Rheumatic Diseases

1. A 56-year-old man presents for evaluation in a primary care clinic. He has a 2-day history of right ankle swelling and pain. He reports experiencing discomfort with ambulation and when driving an automobile. On further questioning, he denies experiencing a recent trauma, although he does recall spraining his ankle approximately 10 years ago. On examination, the patient’s temperature is 99.9° F (37.7° C). His right ankle is warm to palpation and reveals an effusion. With passive range of motion of the right ankle, significant pain is elicited.

Which of the following is the most appropriate step to take next in the treatment of this patient?

A. Check the serum uric acid level; if elevated, initiate therapy with indomethacin and colchicine
B. Obtain a plain radiograph of the right ankle to assess for structural damage or chondrocalcinosis
C. Perform arthrocentesis of the right ankle, with analysis of the synovial fluid
D. Treat with ibuprofen and have the patient return to clinic in 1 week if his symptoms do not improve

Key Concept/Objective: To understand the importance of synovial fluid analysis in the evaluation of a patient with acute monoarthritis

In this patient, the acute onset of symptoms, low-grade fever, and lack of trauma warrant a prompt evaluation; empirical therapy will not provide a definitive diagnosis and could potentially result in a serious illness, such as septic arthritis, being missed. Joint aspiration should be performed with aseptic technique as a part of the evaluation of every case of acute monoarthritis. Analysis of the synovial fluid includes a WBC count and differential, appropriate cultures and stains for microorganisms, and polarized-light microscopy. The WBC count in the synovial fluid is useful in distinguishing inflammatory from noninflammatory arthritis: levels greater than 2,000/mm3 are consistent with inflammation. Patients with crystal-induced arthritis usually have counts in excess of 30,000/mm3. The finding of monosodium urate or calcium pyrophosphate dihydrate crystals on polarizedlight microscopy is pathognomonic for gout and pseudogout, respectively; the absence of crystals does not exclude these diagnoses. The serum level of uric acid is of little use in diagnosing gouty arthritis. Twenty percent of patients with gout have normal uric acid levels, and most persons with elevated levels never develop gouty arthritis. Plain radiography is most useful in patients with significant trauma that suggests the possibility of fracture, in those who experience a sudden loss of function, and in those with symptoms that do not improve despite appropriate treatment. This patient did not have any recent trauma and was still able to bear weight (although it did cause pain). Chondrocalcinosis would suggest the diagnosis of pseudogout, but the most appropriate initial evaluation of a patient with a monoarticular arthritis is arthrocentesis. (Answer: C—Perform arthrocentesis of the right ankle, with analysis of the synovial fluid)

2. Which of the following statements regarding immunologic tests is true

A. As a class, immunologic tests are highly sensitive and specific
B. The use of arthritis panels, in which many serologic tests are bundled together, can increase the likelihood of diagnosing a rheumatic disease
C. Low titers of antinuclear antibody (ANA) are uncommon in young women
D. Rheumatoid factor positivity in healthy persons increases with age

Key Concept/Objective: To understand that immunologic tests are not useful as screening tests

As a class, immunologic tests have low specificity and only moderate sensitivity. They are more expensive than other clinical laboratory tests, and the results are less reproducible. Immunologic tests should never be used as screening tests; their greatest utility occurs when the pretest probability of disease is high. The misuse of immunologic tests frequently confounds the diagnosis and leads to unnecessary rheumatology referrals. The use of socalled arthritis panels, in which many serologic tests are bundled together, increases the likelihood of an abnormal test result occurring in a patient without rheumatic disease; such panels should be avoided. It is common for young women to test positive for ANA; approximately 32% of young women will test positive for ANA at low titers. A positive ANA in and of itself is by no means diagnostic of systemic lupus erythematosus. The probability of testing positive for rheumatoid factor increases with age even in healthy persons. Additionally, conditions other than rheumatoid arthritis can be associated with elevations in rheumatoid factor; because of this, a positive test result for rheumatoid factor is not diagnostic of rheumatoid arthritis. A careful and detailed history is the most important part of the evaluation of a patient with arthritis. Laboratory findings should be evaluated in the context of the information obtained by a detailed history and physical examination. Diagnoses of rheumatic diseases should not be based solely on the findings of immunologic tests. (Answer: D—Rheumatoid factor positivity in healthy persons increases with age)

3. A 56-year-old woman with a history of rheumatoid arthritis presents to clinic with symmetrical pain and swelling of her wrists and metacarpophalangeal joints. She reports morning stiffness, and she notes that it now takes her 4 hours to “loosen up” in the morning. Recently, this has caused her much distress, because she has had increasing difficulty with bathing and dressing herself. The patient has been treated with ibuprofen in the past and expresses concern that her condition will continue to decline.

Of the following, which is the most appropriate statement to make to this patient at this time?
A. “Your disease will likely progress and further limit your function regardless of what we do at this point.”
B. “I recognize your concern; we have very effective treatments available that may prevent progression of your disease and help you regain function.”
C. “I’ll have you talk with our social worker about getting a home health nurse to assist you at home.”
D. “We now have effective medications that can cure rheumatoid arthritis; don’t worry.”

Key Concept/Objective: To recognize the importance of patient education regarding the effectiveness of treatment as a critical component in the care of the patient with a rheumatic disease

Patients who present with complaints of joint pain often express the opinion that nothing can be done for their arthritis. One of the most important things the physician can do is to correct this misconception. It is important that the patient understands that treatment greatly improves the condition of most patients with arthritis. Educating the patient about the effectiveness of treatment is the first step toward a successful outcome. It is important to provide realistic goals regarding the effectiveness of treatment options. Very effective therapies for rheumatoid arthritis have been available for 20 years, such that long-term outcome has distinctly improved. New treatments with biologic agents that block the inflammatory cytokines tumor necrosis factor–a and interleukin-1 are very effective, have a rapid onset of action, and prevent radiographic progression of disease. To date, there is no therapy available that is curative of rheumatoid arthritis. This case illustrates the importance of obtaining a functional assessment as a part of the history. After pain, loss of function is of the greatest concern to the patient. Patients with significant functional impairment should be asked more detailed questions about routine activities of daily living. (Answer: B—“I recognize your concern; we have very effective treatments available that may prevent progression of your disease and help you regain function.”)

4. A 48-year-old man with a history of rheumatoid arthritis presents to clinic complaining of left wrist pain and swelling. He has been maintained on prednisone and methotrexate. The patient expresses frustration with this flare, because he had been doing very well the past few months. Vital signs are significant for a temperature of 100.6° F (38.1° C).

Which of the following statements regarding the evaluation of this patient is most accurate?
A. The affected joint should be examined; the examination should include appropriate maneuvers in an attempt to reproduce the patient complaint
B. Frank redness of the skin overlying the left wrist is always present if the pain is secondary to inflammation
C. Increased temperature of the skin overlying the left wrist is common in inflammatory arthritis and is best detected by palpation with the palms
D. Arthrocentesis of the left wrist is not indicated, because the patient is known to have rheumatoid arthritis

Key Concept/Objective: To understand the components and findings of the joint examination in a patient with inflammatory arthritis

By looking at and palpating the joints, the physician can identify the exact anatomic structures that are the source of the patient’s pain and decide whether the pain is caused by inflammation. A goal of the examination is to reproduce the patient’s pain, either by motion of the joint or by palpation. Frank redness of the skin overlying a joint is unusual; however, increased temperature, best detected by palpation with the backs of the fingers (not the palms), is common and, when present, indicates inflammation. Palpation for tenderness may reveal whether the problem lies within the joint or is discretely localized to an overlying bursa or tendon sheath. Arthrocentesis of the left wrist should be performed as part of the evaluation of this patient. When patients with established rheumatoid arthritis have fever and an apparent flare, joint infection should be excluded by joint aspiration because septic arthritis occurs more frequently in such patients. (Answer: A—The affected joint should be examined; the examination should include appropriate maneuvers in an attempt to reproduce the patient complaint)

For more information, see Ruddy S: 15 Rheumatology: I Introduction to the Rheumatic Diseases. ACP Medicine Online (www.acpmedicine.com). Dale DC, Federman DD, Eds. WebMD Inc., New York, March 2005

Rheumatoid Arthritis

5. A 41-year-old woman comes in for a checkup. You diagnosed her with rheumatoid arthritis (RA) several years ago when she presented with bilateral metacarpophalangeal joint swelling with stiffness and fatigue. The course of this patient’s disease has been mild, and the patient has been maintained on nonsteroidal anti-inflammatory drugs (NSAIDs) and methotrexate therapy. Today she is doing well; she has minimal pain and functional impairment. She asks you about the cause of RA. This stimulates you to read about current evidence regarding the pathogenesis of this illness.

Which of the following statements regarding the pathogenesis of RA is false?
A. Damage to bone and cartilage by synovial tissue and pannus is mediated by several families of enzymes, including serine proteases and cathepsins
B. IgG rheumatoid factor is most commonly detected in patients with RA
C. Interaction of rheumatoid factors with normal IgG activates complement and thereby starts a chain of events that includes production of anaphylatoxins and chemotactic factors
D. Although many cytokines are involved in the pathogenesis of RA, tumor necrosis factor–a (TNF-a) and interleukin-1 (IL-1) are major pathogenic factors

Key Concept/Objective: To understand the pathogenesis of RA

Damage to bone and cartilage by synovial tissue and pannus is mediated by several families of enzymes, including serine proteases and cathepsins. The most damaging enzymes are the metalloproteinases (e.g., collagenase, stromelysin, and gelatinase) and cathepsins (especially cathepsin K), which can degrade the major structural proteins in the joint. IgM rheumatoid factor is most commonly detected; IgG and, less frequently, IgA rheumatoid factors are also sometimes found. The presence of IgG rheumatoid factor is associated with a higher rate of systemic complications (e.g., necrotizing vasculitis). Interaction of rheumatoid factors with normal IgG activates complement and thereby starts a chain of events that includes production of anaphylatoxins and chemotactic factors. Macrophageand fibroblast-derived cytokines (e.g., IL-1, IL-6, TNF-a, and granulocyte-macrophage colony-stimulating factor) are abundantly expressed in the rheumatoid joint. Although many of these cytokines are involved in the pathogenesis of RA, TNF-a and IL-1 are major pathogenic factors; both can induce synoviocyte proliferation, collagenase production, and prostaglandin release. (Answer: B—IgG rheumatoid factor is most commonly detected in patients with RA)

6. A 29-year-old woman visits your office for the evaluation of painful hand swelling. She was in her usual state of health until 2 months ago, when she began to notice moderate morning hand pain. The pain seems to be worsening. She states that her hands are stiff and painful each morning, but they tend to improve over the course of the day. Her pain is localized to the knuckles of both hands. She denies having any rash, difficulty breathing, fevers, or other joint pains. The only notable finding on her physical examination is boggy edema and tenderness to palpation of her metacarpophalangeal joints and proximal interphalangeal joints. She has no wrist pain or deformity. Laboratory test results are normal except for a mild normocytic anemia and an elevated CRP. The patient tests negative for serum rheumatoid factor.

Which of the following statements regarding the diagnosis of RA is true?

A. The typical initial presentation of RA is isolated arthritis of large joints such as the knee or ankle
B. The diagnosis of RA is confirmed by joint stiffness, which is a specific finding for the illness
C. RA tends to cause marked erythema of the involved joints
D. Classically, RA is a symmetrical arthritis

Key Concept/Objective: To know the typical features of RA

Small joints of the hands and feet are usually involved at the outset, although large joints (e.g., knees and ankles) are sometimes affected first. In about 10% of cases, monoarthritis of a large joint can presage progression to polyarticular RA. Most patients experience some degree of joint stiffness, especially in the morning after awakening, which may accompany or precede joint swelling or pain. These symptoms are hallmarks of disease activity and help distinguish RA from noninflammatory diseases such as osteoarthritis. However, joint stiffness and swelling are not specific for RA and can occur with other types of inflammatory arthritis. Unlike acute inflammatory arthritides (e.g., gout or septic arthritis), RA tends not to cause marked erythema, and swelling usually does not extend far beyond the articulation. Classically, RA is symmetrical. When RA is progressive and unremitting, nearly every peripheral joint may eventually be affected, although the thoracic, lumbar, and sacral spine are usually spared. (Answer: D—Classically, RA is a symmetrical arthritis)

7. A 48-year-old female patient of yours with moderately severe RA presents for a scheduled visit. She is very satisfied with her current therapy and feels that joint pains, swelling, and stiffness have all improved over the past 3 months. Her energy level has also improved, and she has recently planted a large flower garden. Her only complaint today is that she can’t “catch her breath” when she works in her garden. Her shortness of breath is worsened by exertion, and she now states that she experiences shortness of breath while ambulating in her house. Over the past week, she has developed pain in her right chest; the pain worsens with exertion or with deep inspiration. Physical examination is noteworthy for decreased breath sounds, decreased fremitus, dullness to percussion, and a pleural rub of the right basilar lung field. Chest radiography confirms the diagnosis of rheumatoid lung disease.

Which of the following statements regarding rheumatoid lung disease is true?
A. The most common form of lung involvement is pleurisy with effusions
B. Rheumatoid effusions typically have a glucose concentration of greater than 50 mg/dl
C. RA is not a reported cause of cavitary lung disease
D. Rheumatoid lung disease with fibrosis typically causes an obstructive ventilatory defect with a decreased carbon dioxide diffusion rate

Key Concept/Objective: To know the key features of rheumatoid lung disease

The most common form of lung involvement in RA is pleurisy with effusions. Evidence of pleuritis is often found at postmortem examination, but symptomatic pleurisy occurs in fewer than 10% of patients. Clinical features include gradual onset and variable degrees of pain and dyspnea. The effusions generally have protein concentrations greater than 3 to 4 g/dl, as well as glucose concentrations lower than 30 mg/dl; the latter finding has been ascribed to a primary defect in glucose transport. Rheumatoid nodules occur in the pulmonary parenchyma and on the pleural surface. They range in size from just detectable to several centimeters in diameter. They may be single or multiple. At times, the nodules cavitate. Such nodules can be difficult to distinguish radiologically from tuberculous or malignant lesions and often require further evaluation, including biopsy. Progressive, symptomatic interstitial pulmonary fibrosis that produces coughing and dyspnea in conjunction with radiographic changes of a diffuse reticular pattern (i.e., honeycomb lung) is usually associated with high titers of rheumatoid factor. The lesion is histologically indistinguishable from idiopathic pulmonary fibrosis. Chest radiographs show pleural thickening, nodules, diffuse or patchy infiltrates, and a restrictive ventilatory defect that is characterized by a decreased carbon dioxide diffusion rate. (Answer: A—The most common form of lung involvement is pleurisy with effusions)

8. A 35-year-old woman returns for a follow-up visit. The patient has been receiving a cyclooxygenase-2 (COX-2) selective NSAID for RA, with only minimal improvement in her symptoms. She continues to have significant pain and morning stiffness in her hands and wrist. Her fatigue is stable and without improvement. She has been reading about the many available therapies for RA and feels that she now needs additional therapy. You explain that the NSAID was only the starting point for her medical therapy and that you agree that it is time to change her therapy.

Which of the following statements regarding accepted medical therapy of RA is false?
A. An acceptable escalation in this patient’s therapy would be to begin methotrexate at the recommended starting dose
B. The antimalarial drug hydroxychloroquine is useful as early secondline therapy for RA
C. An acceptable escalation in this patient’s therapy would be to replace her current therapy with infliximab monotherapy
D. The conventional wisdom is that glucocorticoids neither alter the course of the disease nor affect the ultimate degree of damage to joints or other structures

Key Concept/Objective: To understand medical therapy for RA

Advancement from NSAIDs to second-line agents is recommended if (1) symptoms have not improved sufficiently after a short trial of NSAIDs, (2) the patient has aggressive seropositive disease, or (3) there is radiographic evidence of erosions or joint destruction. The trend today is for more aggressive treatment, and the majority of patients require additional pharmacotherapy. Most patients require rapid advancement from NSAIDs to a second-line agent, most often methotrexate. In the United States, most rheumatologists prefer to increase the methotrexate dosage rapidly to 20 to 25 mg/wk and then add another agent within 2 to 3 months if necessary. The antimalarial drug hydroxychloroquine is useful as early second-line therapy for RA. Its response rate is lower than that of methotrexate, and less improvement is seen; however, its relative safety makes it an ideal choice for patients with mild early disease or as an additive agent in combination therapy. Infliximab is used in combination with methotrexate; this appears to permit long-term use of infliximab with less formation of neutralizing antibodies. Infliximab is administered by intravenous infusion; the recommended dose is 3 to 10 mg/kg every 8 weeks. As with etanercept and all TNF inhibitors, the drug must be used with care in the presence of infections. The conventional wisdom is that glucocorticoids neither alter the course of the disease nor affect the ultimate degree of damage to joints or other structures. (Answer: C—An acceptable escalation in this patient’s therapy would be to replace her current therapy with infliximab monotherapy)

9. A 60-year-old man is diagnosed with RA after several months of joint pain, swelling, and stiffness. His disease has been progressing and involves numerous joints. The patient tests positive for rheumatoid factors and rheumatoid nodules. He is concerned about his prognosis.

Which of the following is associated with a favorable course for a patient with RA?
A. Age greater than 40 years
B. Acute onset in a few large joints
C. Insidious onset of disease
D. Positive rheumatoid factor

Key Concept/Objective: To know the major prognostic factors in RA

In approximately 75% of patients with RA, the disease waxes and wanes in severity over a number of years. Other patients have complete remission. A favorable course and long remissions are associated with age less than 40 years, acute onset restricted to a few large joints, disease duration less than 1 year, and negative test results for rheumatoid factors. An unfavorable prognosis is associated with insidious onset, constitutional symptoms, the rapid appearance of rheumatoid nodules, the appearance of bone erosions early in the course of disease, and high titers of rheumatoid factors. Patients with the most aggressive form of the disease experience a significant loss in quality of life and a shortened life expectancy. Early aggressive management with disease-modifying agents is clearly indicated for patients with an unfavorable prognosis. (Answer: B—Acute onset in a few large joints)

10. A 35-year-old woman has been seeing you for treatment for RA for several years. Her disease is currently well controlled, but she is anxious about her future. She has read extensively about RA and recently learned that patients with the disease die at a younger age than other persons. She asks you about this and about which diseases most commonly cause death in patients with RA.

What is the most common cause of death in patients with RA?
A. Cardiovascular disease
B. Leukemia
C. Infection
D. Non-Hodgkin lymphoma

Key Concept/Objective: To understand that mortality is higher in patients with RA and to be able to identify the leading causes of death in patients with RA

Patients with RA die at earlier ages than those without the disease. The leading cause of death in patients with RA is cardiovascular disease (40% to 45% of deaths); this increase in cardiovascular mortality may be related to the chronic inflammation caused by the disease and to the potential for vascular disease associated with treatments such as glucocorticoids. Malignancy accounts for 15% of deaths in these patients; infections account for
10%. There is an increased incidence of lymphoproliferative diseases such as non-Hodgkin lymphoma and Hodgkin disease in patients with RA. (Answer: A—Cardiovascular disease)

11. A 34-year-old woman with a 5-year history of rheumatoid arthritis presents for routine follow-up. She has been taking sulfasalazine, 1 g b.i.d., and naproxen, 500 mg b.i.d. She complains of 1 to 2 hours of morning stiffness and mild swelling of multiple PIP and MCP joints. Physical examination (in early afternoon) reveals mild synovitis of the MCPs and PIPs of both hands and difficulty making a full fist. She also has synovitis of multiple MTP joints. X-rays demonstrate small erosions in the joints of the hands and feet that seem to have progressed since last year.

What therapy would you recommend for this patient’s arthritis?
A. Continue the current therapy
B. Increase sulfasalazine to 3 g/day
C. Add prednisone, 5 mg/day
D. Add hydroxychloroquine, 400 mg/day
E. Add oral methotrexate, 7.5 mg/wk

Key Concept/Objective: To understand the treatment of progressive rheumatoid arthritis

To prevent the development of erosive joint changes, the standard of treatment for rheumatoid arthritis has become much more aggressive. This patient on sulfasalazine has developed further x-ray changes, and the next step for most rheumatologists would be to add methotrexate to the regimen to gain more control of the synovitis that is damaging her joints. The dose could be increased to as much as 25 mg/wk, if needed, to control her disease. More than 2 g/day of sulfasalazine is rarely more effective but is potentially more toxic. Low-dose prednisone may help her symptoms but would not affect the disease process. Finally, hydroxychloroquine, although safe, has less disease-modifying power than methotrexate. Recently, the combination of methotrexate, sulfasalazine, and hydroxychloroquine has been shown to be an effective combination in resistant disease and could be used if the addition of methotrexate is insufficient. (Answer: E—Add oral methotrexate,
7.5 mg/wk)

12. A 62-year-old woman comes to clinic complaining of right eye pain and redness. It has been present for several weeks and is getting worse. She has a history of rheumatoid arthritis and has been on hydroxychloroquine, 400 mg/day, and prednisone, 5 mg/day, for several years. On examination, the eye is very red, with a violaceous hue to the sclera. Gentle finger pressure over the eyelid onto the globe is painful.

Which of the following should be the next step in the care of this patient?
A. Add an ocular lubricant to her regimen
B. Prescribe corticosteroid ocular drops
C. Stop the hydroxychloroquine immediately
D. Increase the prednisone to 20 mg/day
E. Call for an ophthalmology appointment

Key Concept/Objective: To be able to recognize serious eye disease in rheumatoid arthritis

Patients with rheumatoid arthritis may have a variety of eye problems, including dry eye, episcleritis, and scleritis. Dry eye is rarely serious and is treated with eyedrops and lubricants. Episcleritis is inflammation of superficial vessels and is generally not a threat to vision; scleritis is caused by inflammation of the deeper vessels and can lead to loss of vision. Differentiation of the two is based on the more violaceous hue of the sclera in scleritis—caused by inflammation around the sclera vessels—and pain on pressure over the closed lid onto the globe, which is not seen in episcleritis. Confirmation can be done by slit-lamp examination. Scleritis comes in different forms, and treatment decisions are best made in conjunction with an ophthalmologist. The most serious form of scleritis can lead to scleromalacia perforans, which usually leads to blindness in the affected eye. Hydroxychloroquine can cause eye disease, notably retinopathy, but there are no superficial manifestations. (Answer: E—Call for an ophthalmology appointment)

13. A 55-year-old man comes to your clinic with a persistent cough. He had a viral syndrome 3 weeks ago that has cleared, but he continues to have a nonproductive cough. Past medical history is significant for seropositive rheumatoid arthritis and a 25-pack-year history of smoking. Current medications include leflunomide, 10 mg/day, and prednisone, 5 mg/day. Physical examination is significant for mild ulnar deviation of the fingers and fibular deviation of the toes, but little active synovitis. Rheumatoid nodules are present over the extensor surface of both forearms near the elbows. Chest x-ray reveals a 2 cm × 2 cm pulmonary nodule in the right upper lobe but is otherwise normal. Old films are not available.

Which of the following should be the next step in the care of this patient?
A. Reassure and treat symptomatically
B. Increase the leflunomide to 20 mg/day
C. Repeat the chest x-ray in 3 months
D. Perform a CT scan to evaluate the lesion further
E. Schedule a transbronchial biopsy

Key Concept/Objective: To understand the evaluation of pulmonary nodules in patients with rheumatoid arthritis

Patients with rheumatoid arthritis, particularly men with subcutaneous nodules who are smokers, are prone to developing rheumatoid nodules in the lung. These lesions are generally well circumscribed. They can be of various sizes, may be single or multiple, and tend to be peripheral in location. Unfortunately, those patients who are at risk for rheumatoid lung nodules are also at risk for lung cancer, and pulmonary nodules in patients with rheumatoid arthritis should be considered potentially malignant. A CT scan of the chest is the most reasonable first step to evaluate location and the presence of adenopathy. In most cases, a biopsy will be necessary for histologic evaluation. (Answer: D—Perform a CT scan to evaluate the lesion further)

14. A 35-year-old woman comes to clinic for follow-up of rheumatoid arthritis and to evaluate a new rash on the lower extremities. She was diagnosed with rheumatoid arthritis 5 years ago on the basis of joint pain and a positive rheumatoid factor, but the rheumatoid factor has been intermittently positive since then. Current medications include methotrexate, 7.5 mg/wk, and ibuprofen, 400 mg t.i.d. Physical examination is significant for the lack of synovitis in the small joints of the hands and feet and the presence of palpable purpura on both lower extremities. Biopsy of the purpura reveals leukocytoclastic vasculitis.

Which of the following would be the most useful serologic test to clarify this patient’s illness?
A. Antineutrophil cytoplasmic antibodies
B. Antinuclear antibody reflexive panel
C. Hepatitis C antibody with PCR if positive
D. Hepatitis B antibodies, including anticore
E. Repeat the testing for rheumatoid factor

Key Concept/Objective: To be able to recognize the mimicking of rheumatoid arthritis by hepatitis C infection

Patients with hepatitis C infection may have polyarthralgias or polyarthritis that can resemble rheumatoid arthritis. To make matters even more problematic, rheumatoid factor is present in many patients with hepatitis C, especially in the setting of mixed cryoglobulinemia. The rheumatoid factor, as part of the cryoglobulin, may not be present in the serum if it is collected and allowed to clot at room temperature. Cryoglobulins will aggregate and clot if subjected to temperatures generally lower than 100.4° F (38° C). If rheumatoid arthritis is suspected, the specimen should be allowed to clot in a 38° C water bath and then checked for rheumatoid factor. Patients with hepatitis C should in general avoid potentially hepatotoxic drugs such as methotrexate. (Answer: C—Hepatitis C antibody with PCR if positive)

15. A 45-year-old woman with a 10-year history of rheumatoid arthritis comes to clinic with a 3-day history of right knee pain and swelling. She cannot bend the knee without severe discomfort. She has also noted a mild increase in pain and swelling of the small joints of her hands and feet. Current medications include methotrexate, 15 mg/week, prednisone, 5 mg/day, and ibuprofen, 600 mg t.i.d. Physical examination reveals ulnar deviation of the fingers, with 1+ synovitis of the MCPs and PIPs, hammer toe deformities, and fibular deviation of the toes, also with 1+ synovitis. The right knee has a significant effusion, is erythematous, and is warm to the touch. She resists attempts to flex and extend the knee. X-rays of the knee show mild, diffuse joint-space narrowing, unchanged from films taken last year.

Which of the following should be the next step in the care of this patient?
A. Increase prednisone to 30 mg/day for 1 week, then taper
B. Increase the dose of methotrexate to 17.5 mg/week
C. Order an MRI to evaluate the knee for internal derangement
D. Perform arthrocentesis and give colchicine, 0.6 mg t.i.d.
E. Perform arthrocentesis and then admit for I.V. antibiotics

Key Concept/Objective: To be able to recognize septic arthritis in a patient with underlying rheumatoid arthritis

Patients with rheumatoid arthritis are at increased risk for septic arthritis. The most common joint affected by septic arthritis is the knee. Clues to an underlying septic arthritis in this patient include severe joint pain (rare in rheumatoid joints), erythema (also rare), and a joint that is much more symptomatic than the rest. Patients with rheumatoid arthritis may not have the usual systemic symptoms of fever and chills because of the anti-inflammatory medications that are used to treat the chronic arthritis. In general, it is important to have a high degree of suspicion; when in doubt, rule out septic arthritis. The mortality is as high as 20% for septic monoarthritis in patients with underlying rheumatoid arthritis and up to 50% if more than one joint is infected. (Answer: E—Perform arthrocentesis and then admit for I.V. antibiotics)

For more information, see Firestein GS: 15 Rheumatology: II Rheumatoid Arthritis. ACP Medicine Online (www.acpmedicine.com). Dale DC, Federman DD, Eds. WebMD Inc., New York, July 2004

Seronegative Spondyloarthropathies

16. A 27-year-old man comes to your office asking that you evaluate him for the possibility of having ankylosing spondylitis. His older brother has recently been diagnosed with ankylosing spondylitis, and he has learned on the Internet that ankylosing spondylitis runs in families. He is completely asymptomatic, and his physical examination, including a careful examination of his back, sacroiliac joints, and heart, is unremarkable. You consider ordering a test to assess for the presence of the HLA-B27 allele.

In which patients is testing for HLA-B27 indicated?
A. Any male patient under the age of 35 years who presents with chronic back pain (i.e., pain persisting over 6 months) should be assessed for HLA-B27 as part of the routine workup
B. HLA-B27 testing is never indicated
C. All first-degree relatives of patients with a confirmed diagnosis of ankylosing spondylitis should be screened for possible disease with HLA-B27 testing
D. Only those patients whose clinical presentation and examination are consistent with ankylosing spondylitis but whose radiographic testing is negative should undergo HLA-B27 testing

Key Concept/Objective: To understand the role of HLA-B27 in the diagnosis of ankylosing spondylitis

The diagnosis of ankylosing spondylitis is based on the following modified New York criteria: (1) low back pain of at least 3 months’ duration that is alleviated with exercise and is not relieved by rest; (2) restricted lumbar spinal motion; and (3) decreased chest expansion relative to normal values for age and sex. In addition, the patient must have definitive radiographic evidence of sacroiliitis. HLA-B27 is not required for the diagnosis. The lack of specificity of HLA-B27 in asymptomatic patients precludes its use in this patient. Infrequently, a patient can present with clinical stigmata of disease without radiographic evidence of disease. After other primary disease processes have been ruled out, such as reactive arthritis, psoriasis, or inflammatory bowel disease, it is reasonable to test for HLA-B27. In this subgroup of patients, follow-up sacroiliac radiographic abnormalities will eventually evolve; this may take as long as 10 years. (Answer: D—Only those patients whose clinical presentation and examination are consistent with ankylosing spondylitis but whose radiographic testing is negative should undergo HLA-B27 testing)

17. A 28-year-old white man presents as a walk-in patient to your clinic. He reports a 5-day history of rightsided knee pain and bilateral ankle pain. He also thinks his back has been “a little stiff.” He reports no trauma or previous joint ailments, nor does he have a family history of hematologic diseases or joint disorders. He reports mild fever subjectively, no visual changes or eye pain, and very mild and intermittent dysuria. On physical examination, the patient is found to have mild effusion in his right knee, without overt inflammation, and bilateral tenderness of his Achilles tendons. Genital examination is significant for shallow ulcerations on the glans of the penis upon foreskin retraction.

Which of the following statements regarding this patient is true?
A. The diagnosis of reactive arthritis is highly unlikely given the absence of evidence of uveitis or urethritis in either the history or the physical examination
B. Normal gut flora have a prominent role in the pathogenesis of this disease process
C. A careful sexual history should be obtained, and testing for Chlamydia trachomatis (i.e., culture or molecular probe assay) should be completed
D. Oral ciprofloxacin should be prescribed for this patient

Key Concept/Objective: To understand the pathogenesis and the classic presentation of reactive arthritis

The presence of an asymmetrical arthritis and balanitis circinata in this patient is highly suggestive of reactive arthritis, one of the seronegative spondyloarthropathies. Reactive arthritis was originally defined as the triad of nongonococcal urethritis, conjunctivitis, and arthritis. It is now recognized that most patients present with arthritis alone and have no clinical evidence of urethritis or conjunctivitis. Reactive arthritis provides the strongest evidence of bacterial pathogenesis in the spondyloarthropathies. Enteric infections by pathogens such as Shigella flexneri, Salmonella (many species), Yersinia enterocolitica, Y. pseudotuberculosis, and Campylobacter jejuni have all been implicated as triggers of the disease (epidemic or postenteric form), especially in HLA-B27–positive persons. Similarly, sexually acquired infections with Chlamydia trachomatis and perhaps Ureaplasma urealyticum may cause reactive arthritis (endemic or postvenereal form). HLA-B27 is found in 63% to
75% of patients with both forms of reactive arthritis and confers a relative risk of approximately 37. Because the symptoms associated with Chlamydia genital infections can be subtle, a careful sexual history should be obtained; testing for Chlamydia should be pursued if a history of venereal exposure or genitourinary symptoms is obtained. Early treatment of genitourinary infections with appropriate antibiotics (tetracycline or erythromycin) has been shown to reduce the likelihood of subsequent reactive arthritis; however, even early antibiotic use in patients with gastroenteritis does not appear to prevent reactive arthritis. A blinded, placebo-controlled trial of the use of tetracycline for the treatment of reactive arthritis demonstrated that the duration of disease was shortened only in patients who had Chlamydia-induced disease. (Answer: C—A careful sexual history should be obtained, and testing for Chlamydia trachomatis [i.e., culture or molecular probe assay] should be completed)

18. A 49-year-old white male patient of yours comes to you with hand pain of new onset. His medical history is significant only for hypertension, which is well controlled. He has experienced hand pain and swelling intermittently for the past 6 months, but this has hardly interfered with his daily activities. He indicates that the second and third distal interphalangeal joints on his right hand and the fourth distal interphalangeal joint on his left hand give him the most trouble. He reports no trauma or repetitive activities. Physical examination reveals fingers that are markedly swollen and inflamed but are remarkably nontender on palpation. Range of motion is preserved. Skin examination reveals no rash; however, the scalp has several small areas of silver scaling.

Regarding this patient, which of the following statements is true?
A. This patient likely has rheumatoid arthritis, given the joints that are involved
B. Psoriatic arthritis is highly unlikely, given the fact that there is no history of psoriasis and the lack of extensive skin changes consistent with psoriasis
C. Radiographic changes characteristic of this patient’s condition include a pencil-in-cup appearance and periostitis
D. Gold, penicillamine, and hydoxychloroquine are considered first-line agents in the treatment of this patient

Key Concept/Objective: To know the classic presentation of psoriatic arthritis and to understand the radiographic changes and treatment options

An inflammatory arthropathy attributable to psoriasis appears in 5% to 7% of patients with the skin disease, especially in those whose nails are affected. In general, there is little relation between joint disease and the severity of skin involvement. In fact, psoriatic skin lesions may be found only after careful scrutiny of scalp, umbilicus, or gluteal regions, and nail pitting or other changes may be the only clues supporting a diagnosis of psoriatic arthritis. Asymmetrical oligoarthritis of both small and large joints is the most common form of psoriatic arthritis. Involvement of the distal interphalangeal joints and sausageshaped toes or fingers are highly suggestive signs. A disparity is often noted between clinical appearance and subjective symptoms; overtly involved joints may be largely asymptomatic, unlike the concordance usually found in rheumatoid arthritis. A characteristic change is the whittling of the distal ends of phalanges, giving the joints a so-called pencilin-cup appearance, which is radiographically distinctive for psoriatic arthritis. Periostitis, bony erosions, and joint effusions are also common and so are diagnostically useful. (Answer: C—Radiographic changes characteristic of this patient’s condition include a pencil-in-cup appearance and periostitis)

19. A 21-year-old man who has experienced low back pain for the past 5 months comes to the clinic for evaluation. The pain wakes him at night, but by getting up and moving around he is able to go back to sleep. He has stiffness in the arm that lasts for 1 hour or so and is lessened by a hot shower. He also complains of right groin pain, which has the same character as the back discomfort. His only previous musculoskeletal problem was a prolonged bout of Achilles tendonitis 3 months ago. His father has had back problems for years. Ibuprofen, 1,200 mg/day, gives him little relief. Recent x-rays of the lumbar spine and pelvis were interpreted as being normal.

Of the following, which is the best step to take next in the management of this patient?
A. Determine erythrocyte sedimentation
B. Evaluate for the presence of HLA-B27 gene
C. Perform bone scan of the spine and pelvis
D. Perform CT scan of the sacroiliac joints
E. Start a regimen of indomethacin, 50 mg t.i.d.

Key Concept/Objective: To understand the diagnosis of ankylosing spondylitis

This patient has classic inflammatory low back pain. In addition, he appears to have involvement of the hip and enthesitis (inflammation where connective tissue inserts into bone) of the heel. Analgesic doses of ibuprofen are minimally effective because antiinflammatory dosages of NSAIDs (i.e., dosages in the upper level of the dosing range) are usually necessary to achieve any symptomatic improvement in ankylosing spondylitis. Indomethacin at a dosage of 50 mg t.i.d. would be anti-inflammatory, but a response is not diagnostic. Determination of the erythrocyte sedimentation rate may confirm the inflammatory nature of the symptoms; the presence of HLA-B27 is not diagnostic but might suggest the presence of a spondyloarthropathy. A bone scan is not specific, and the sacroiliac joints normally take up the radiotracer used in a bone scan. A CT scan of the sacroiliac joints can demonstrate early bony and cartilage changes not visible on regular x-rays—in this case, CT would be the best method of diagnosis. (Answer: D—Perform CT scan of the sacroiliac joints)

20. A 44-year-old woman complains of pain and swelling of the right ankle and foot. The symptoms have been present for 4 weeks and are generally worse in the morning. She had an episode of right knee swelling and pain 2 years ago that seemed to have responded to a course of NSAIDs. Her medical history is significant for hypertension, treated by hydrochlorothiazide, 25 mg/day, and mild scalp psoriasis, treated by tar shampoo. On physical examination, the blood pressure is 130/84 mm Hg, there are several small patches of hyperkeratosis in the scalp, the right ankle has a moderate effusion, and there is dactylitis of the second and fourth toes on the right foot.

Which of the following should be the next step in treating this patient?
A. Perform arthrocentesis of the right ankle and send fluid for culture
B. Perform arthrocentesis of the right ankle and send fluid for crystal analysis
C. Order x-rays of the foot and ankle to look for evidence of joint erosions
D. Test for rheumatoid factor to evaluate for possible early rheumatoid arthritis
E. Start the patient on piroxicam, 20 mg/day, and have her return in 1 month

Key Concept/Objective: To be able to recognize psoriatic arthritis

This patient has classic psoriatic arthritis. The diagnosis is based on clinical presentation, because there are no definitive diagnostic tests. The oligoarticular nature of this patient’s arthritis, along with the presence of dactylitis (sausage digits) and scalp psoriasis, makes the differential diagnosis short indeed. Piroxicam is a potent anti-inflammatory, and use of this agent is a reasonable first step in addressing the arthritis. (Answer: E—Start the patient on piroxicam, 20 mg/day, and have her return in 1 month)

21. A 30-year-old man who recently experienced an attack of uveitis is referred by an ophthalmologist for evaluation for possible underlying systemic disease. The patient’s episode of uveitis involved the left eye and lasted 3 weeks; the uveitis responded to topical corticosteroids. The patient denies having any pulmonary symptoms, diarrhea, urethritis, peripheral joint pain or swelling, or recent low back pain. When he was in his early 20s, he was involved in a car accident and for several years after experienced low back pain. He is an avid soccer player but has had to avoid playing recently because of plantar fasciitis of the right foot. On examination, the eyes are without inflammation, the lungs are clear, there is no peripheral joint swelling and no tenderness over the sacroiliac joints, the Schober test demonstrates 3 cm of distraction, and there is tenderness in the right heel at the insertion of the plantar fascia.

Which of the following would be the most useful step to take next in the evaluation of this patient

A. Posteroanterior and lateral chest x-rays
B. HLA-B27 determination
C. Pelvic outlet view of the sacroiliac joints
D. Lateral x-ray of the heel
E. GI consult for sigmoidoscopy

Key Concept/Objective: To be able to recognize systemic disease underlying uveitis

The differential diagnosis of unilateral uveitis includes ankylosing spondylitis, Reiter syndrome, and inflammatory bowel disease. Many cases, however, are idiopathic. Sarcoidosis typically causes bilateral uveitis. This patient has symptoms that suggest an underlying spondyloarthropathy. The most useful test would be a pelvic outlet view of the sacroiliac joints, particularly given this patient’s history of low back pain. Determination of the presence of HLA-B27 would be useful only to further the suspicion of an underlying spondyloarthropathy. The foot film might demonstrate the presence of enthesitis, but it would not be as diagnostic of spondyloarthropathy as it would be of sacroiliitis. (Answer: C—Pelvic outlet view of the sacroiliac joints)

22. A 48-year-old man with longstanding ankylosing spondylitis is brought to the emergency department after a minor rear-end motor vehicle accident. He is complaining of neck pain. Plain films of the neck demonstrate advanced ankylosing spondylitis with a bamboolike cervical spine. No fracture is seen, and the neurologic examination is normal.

Which of the following would be the most useful step to take next for this patient?

A. Reassure and follow up in 1 week
B. Prescribe a mild muscle relaxant
C. Place in a rigid collar and refer
D. Order an MRI of the cervical spine
E. Place in immediate cervical traction

Key Concept/Objective: To be able to recognize post-trauma cervical fracture in ankylosing spondylitis

Patients with longstanding ankylosing spondylitis and bamboo-type spine are at risk for fracture through the fused disk space. Such a fracture may lead to an unstable spine and myelopathy. This condition is difficult to recognize with plain x-rays, and the best course of action would be to evaluate the cervical spine for the presence of a disk-space fracture through a more sensitive diagnostic approach, such as MRI. If a fracture is demonstrated, immediate neurosurgical or orthopedic spinal surgery consultation is required. (Answer: D—Order an MRI of the cervical spine)

For more information, see Arnett FC: 15 Rheumatology: III Seronegative Spondyloarthropathies. ACP Medicine Online (www.acpmedicine.com). Dale DC, Federman DD, Eds. WebMD Inc., New York, November 2002

Systemic Lupus Erythematosus

23. A 24-year-old woman presents to your clinic as a new patient. She complains of fatigue, and she has experienced a 10 lb weight loss over the past several months. On review of systems, she admits to moderate myalgias and arthralgias. She denies having any rash involving her face, but she has occasionally noted a rash on her hands. She also experiences pain and skin changes in cold weather. She reports some mild dyspnea and pain on inspiration. Blood work reveals a WBC of 2,500 with a relative lymphopenia. The serum antinuclear antibody (ANA) titer is 1:80; the anti–double-stranded DNA antibody assay is negative, as is anti-Smith (anti-Sm) antibody assay; the anti-ribonucleoprotein (anti-RNP) assay is positive with a high titer. Urinalysis is normal.

This clinical picture with the given serologies is most consistent with which of the following rheumatologic disorders?
A. Systemic lupus erythematosus (SLE)
B. Dermatomyositis
C. Mixed connective tissue disease (MCTD)
D. Undifferentiated connective tissue disease (UCTD)

Key Concept/Objective: To understand that lupuslike symptoms may present as part of an overlap syndrome

Some patients have symptoms suggestive of lupus (most commonly, arthritis, pleuritic chest pain, and cytopenia) but lack the specific diagnostic criteria for lupus (e.g., butterfly rash, glomerulonephritis, high-titer anti-dsDNA, or anti-Sm antibody). Other patients have lupuslike symptoms together with findings suggestive of rheumatoid arthritis, dermatomyositis, or scleroderma. Those with no definable serology and a nondescript clinical picture are defined as having UCTD. Other patients have inflammatory myositis, Raynaud phenomenon, and sclerodactyly together with very high titer antibodies to the ribonucleoprotein antigen (U1 RNP) and no anti-DNA or anti-Sm antibody. This set of findings is defined as MCTD. The differentiation of SLE from UCTD, MCTD, and Sjögren syndrome depends on the extent and pattern of different organ involvement (glomerulonephritis is rare in all these disorders except lupus) and on the accompanying serologic abnormalities. This patient presents with several complaints consistent with a connective tissue disease, including serositis, arthralgias, myalgias, and a nonspecific skin rash affecting predominantly the hands. The combination of a low-titer ANA and negative anti-dsDNA and antiSm makes the diagnosis of SLE questionable. More importantly, the presence of high-titer anti-RNP is consistent with the diagnosis of MCTD. (Answer: C—Mixed connective tissue disease [MCTD])

24. A 31-year-old woman comes to your clinic for follow-up. For the past several years, her lupus has been well controlled without systemic medications. She is employed full-time, and she and her husband have been contemplating pregnancy. Last month, however, she presented to your office complaining of fever, severe arthralgias, myalgias, and a diffuse erythematous rash. The pregnancy test was negative. Results of urinalysis and renal function testing were normal. After a failed trial of NSAIDs, you started her on prednisone, 60 mg/day. At follow-up, she reports that all of her symptoms have improved significantly.

Of the following, what is the most appropriate step to take next in the treatment of this patient?
A. Discontinue her steroids and try another trial of NSAIDs
B. Taper her steroids and add high-dose oral calcium and vitamin D
C. Taper her steroids and add high-dose calcium, vitamin D, and a bisphosphonate
D. Discontinue her steroids and switch to oral cyclophosphamide

Key Concept/Objective: To understand the importance of bone-protective therapies and the contraindications to those therapies for patients on long-term steroid regimens

This patient has experienced a flare of her SLE. She had a good response to high-dose corticosteroids and is likely to require steroid therapy for the next several months. High-dose corticosteroid therapy is used for patients with severe systemic symptoms, renal disease, or other visceral disease that is potentially life-threatening. Treatment should be initiated in split doses during the day, maintained for 4 to 6 weeks, and then tapered; too-early reduction in the dosage usually results in recurrence of disease activity. Osteoporosis follows long-term corticosteroid therapy with sufficient frequency that all patients receiving such therapy should receive prophylaxis for this complication. High-dose oral calcium, vitamin D, and a bisphosphonate are the primary preventive measures; estrogen replacement may be considered in postmenopausal women who do not have antiphospholipid antibody. Because this woman desires to become pregnant, she should not be given an oral bisphosphonate. The decision to start an agent such as cyclophosphamide should be considered when the patient is not responding to steroids or has severe renal disease. Furthermore, cyclophosphamide can cause infertility and be teratogenic if used inadvertently during pregnancy. (Answer: B—Taper her steroids and add high-dose oral calcium and vitamin D)

For more information, see Lockshin MD: 15 Rheumatology: IV Systemic Lupus
Erythematosus. ACP Medicine Online (www.acpmedicine.com). Dale DC, Federman DD, Eds. WebMD Inc., New York, December 2002

Scleroderma and Related Diseases

25. A 32-year-old white man with known scleroderma presents for evaluation. On physical examination, you see that his blood pressure is elevated (165/105 mm Hg). The other findings of the physical examination are unchanged from before. The patient’s creatinine level is 1.5 mg/dl (baseline, 0.8 mg/dl); urinalysis shows trace protein and 5 to 10 red cells. You are concerned about the possibility of scleroderma renal crisis.

Which of the following statements regarding this patient is false?

A. The patient should be treated with a thiazide diuretic
B. The patient’s symptoms are associated with microangiopathic hemolytic anemia
C. It is likely that this patient has received high-dose steroids in the past
D. This patient is at risk for oliguric renal failure
E. Unless this patient receives aggressive treatment, his prognosis is poor

Key Concept/Objective: To be able to recognize scleroderma renal crisis and understand its management

Scleroderma renal crisis is a dreaded complication of diffuse scleroderma. It can occur rapidly and is more likely to be seen in patients with rapidly progressive skin disease. Hypertension of new onset, in conjunction with proteinuria and microscopic hematuria, is highly characteristic. In more severe cases, patients present with malignant hypertension and microangiopathic hemolytic anemia. Scleroderma renal crisis was the most common cause of death among patients with diffuse scleroderma until the advent of angiotensin-converting enzyme (ACE) inhibitors. Use of short-acting agents such as captopril is indicated. Because the response to therapy seems to be better when the creatinine level is less than 3 mg/dl, the diagnosis needs to be made promptly, and therapy should be instituted quickly. Recovery of normal renal function has been documented in patients requiring dialysis who received ACE inhibitors. (Answer: C—It is likely that this patient has received high-dose steroids in the past)

26. A 35-year-old woman known to have long-standing scleroderma comes to you complaining of worsening constipation. She has been experiencing constipation for the past several months; recently, her constipation has become associated with abdominal pain and very hard stools. On occasion she has vomited.

Which of the following is NOT a gastroenterologic complication of scleroderma?

A. Pneumatosis intestinalis
B. Esophageal dysmotility, gastroparesis, and intestinal pseudo-obstruction
C. Colonic diverticulosis
D. Sjögren syndrome
E. Achalasia

Key Concept/Objective: To be aware of the different manifestations of scleroderma in the GI tract

Although not completely understood, scleroderma-associated lesions of the GI tract appear to be the result of autonomic nerve dysfunction of the GI tract. In time, this autonomic nerve dysfunction leads to smooth muscle atrophy and eventually irreversible muscle fibrosis of the gut. As a consequence, hypomotility of the esophagus, stomach, and small and large intestine are seen in patients with systemic sclerosis. Esophageal dysmotility is associated with reflux and, eventually, strictures and even changes associated with Barrett esophagus. Involvement of the stomach and small intestine is associated with gastroparesis and pseudo-obstruction, respectively. The presence of wide-mouth diverticula is pathognomonic of scleroderma. The lower two thirds of the esophagus show an absence of peristaltic waves and incompetence of the lower esophageal sphincter. Achalasia is characterized by an increase, not a decrease, in activity of the lower esophageal sphincter. (Answer: E—Achalasia)

For more information, see Moxley G: 15 Rheumatology: V Scleroderma and Related Diseases. ACP Medicine Online (www.acpmedicine.com). Dale DC, Federman DD, Eds. WebMD Inc., New York, March 2004

Idiopathic Inflammatory Myopathies

27. A 34-year-old woman presents to your clinic complaining of weakness of 1 month’s duration and shortness of breath of 2 weeks’ duration. She says her weakness is worse when she tries to comb her hair or when she tries to stand up from a sitting position. She does not have any significant medical history, and she is not taking any medications or over-the-counter drugs. Her family history is unremarkable. On physical examination, the patient has a hyperkeratotic rash on her hands. The strength of her legs and arms is 5/5 distally and 3/5 proximally. A chest x-ray shows the presence of interstitial infiltrates. You suspect her symptoms are caused by an inflammatory myopathy. You order muscle enzyme levels and a muscle biopsy; the findings are consistent with your presumed diagnosis. You also order testing of autoantibody levels to better categorize her disease.

This patient is likely to test positive for which of the following antibodies?

A. Antimitochondrial antibodies
B. Antibodies against aminoacyl-transfer RNA (tRNA) synthetases
C. Antibodies against signal recognition particle (SRP)
D. Antibodies against Mi-2

Key Concept/Objective: To know the antibodies that correlate with different clinical pictures in idiopathic inflammatory myopathies

Autoantibodies to nuclear and cytoplasmic antigens are found in as many as 90% of patients with an inflammatory myopathy. These antibodies are often useful in differentiating inflammatory myopathies from diseases that are not autoimmune disorders. Some of these antibodies are nonspecific and can be seen in several autoimmune disorders. About 25% of patients with inflammatory myositis test positive for antinuclear antibodies. Autoantibodies that are in large part directed against cytoplasmic ribonucleoproteins have been designated as myositis-specific autoantibodies (MSA) and are present in 30% of the patients. These antibodies tend to correlate with some specific clinical presentations, responses to therapy, and prognoses. Three groups of patients can be defined by the MSA specificities. The first group is defined by the presence of antibodies directed against aminoacyl-tRNA synthetases. These patients are generally characterized by an acute onset of muscle disease, with a high incidence of associated interstitial lung disease. They may also have arthritis and a hyperkeratotic rash on the hands, known as mechanic’s hands. This description fits the patient presented in this case. The second group includes patients with anti-SRP antibodies; these patients tend to have an abrupt onset of weakness, and they may have cardiac disease. The third group is identified by the presence of antibodies against Mi-2; these patients have a dermatomyositis with the so-called shawl sign. (Answer: B—Antibodies against aminoacyl-transfer RNA [tRNA] synthetases)

28. A 58-year-old man is seen in your clinic for the first time. He says he has not seen a doctor in 5 years.
He has no significant medical history. He says he has decided to see a doctor because over the past 2 years he has noticed some weakness of his arms and legs. He says these symptoms were not bothering him initially but that, over the past few months, he has noticed more weakness in his left arm. He is not taking any medications. On physical examination, there is no rash; his strength is 5/5 on the right side of his body, 5/5 in his left leg, and 3/5 in his left arm. His distal strength and proximal strength are quite similar. Neurologic examination results are otherwise normal. His creatine kinase (CK) level is moderately elevated.

Which of the following is the most likely diagnosis for this patient?
A. Dermatomyositis
B. Polymyositis
C. Sarcoidosis
D. Inclusion body myositis (IBM)

Key Concept/Objective: To understand the presentation of inclusion body myositis

This patient is a middle-aged man with slow-onset muscle weakness. Dermatomyositis is defined by the presence of an inflammatory myopathy and a characteristic rash. Polymyositis is characterized by weakness that is symmetrical and predominantly proximal, and the clinical course is more aggressive than the one described here. Sarcoidosis can cause a myopathy but usually is accompanied by other manifestations that are absent here. This patient’s symptoms are more consistent with IBM. The pattern of severity of muscle weakness in IBM differs from that seen in other idiopathic inflammatory myopathies. In addition to the presence of proximal weakness, distal muscles may be involved, and in some cases, muscle abnormalities are asymmetrical. Unlike most of the other inflammatory muscle disorders, IBM affects more men than women. Response to treatment is generally poor. Electron microscopy may be required to demonstrate the inclusion bodies that define IBM. (Answer: D—Inclusion body myositis [IBM])

29. A 40-year-old woman with polymyositis comes to your office for an initial visit. She was diagnosed 3 months ago and was placed on prednisone, 1 mg/kg/day. She says her strength has recovered significantly since she started therapy. She comes to visit you because she recently moved to your town. She has no other significant medical history; she has no allergies, and she is taking no medications other than prednisone. She has a family history of diabetes. Her physical examination shows minimal proximal weakness.

Which of the following would be the best therapeutic intervention at this time?
A. Continue steroids at the same dose for 3 more months and then proceed with a slow taper
B. Start tapering the steroids and start methotrexate or azathioprine
C. Discontinue the steroids and start cyclophosphamide
D. Order a muscle biopsy to see if the patient is in remission

Key Concept/Objective: To understand the treatment of idiopathic inflammatory myopathies

Corticosteroids are the mainstay of initial therapy for the inflammatory myopathies. Most patients with documented muscle inflammation should be started on these drugs at relatively high doses (1 mg/kg/day). A standard approach has been to maintain this dosage for up to 3 months or until clinical improvement occurs. After this initial period of high-dose therapy, the dose can be consolidated into a single morning dose and then tapered so that the total daily dose is reduced by 20% to 25% each month; a maintenance dose of 5 to 10 mg daily should be reached in about 6 to 8 months. The addition of second-line drugs to the prednisone regimen is now recommended within the first 3 months of initiating treatment. The most commonly used second-line agents for the treatment of inflammatory myopathy are methotrexate and azathioprine. Azathioprine and methotrexate have similar efficacy in these disorders, and the choice of which to use may depend on tolerability or other comorbid conditions. Cyclophosphamide may be useful in the treatment of the antisynthetase syndrome with interstitial lung disease and in children with vasculitisrelated complications of dermatomyositis. There is no indication in this case for the use of cyclophosphamide. After the diagnosis is made and the therapy is started, the response to therapy in patients with idiopathic inflammatory myopathies is followed clinically and with assessment of muscle enzyme levels; a muscle biopsy is usually not necessary in this setting. (Answer: B—Start tapering the steroids and start methotrexate or azathioprine)

30. A 35-year-old woman comes to your office complaining of weakness and fatigue that has lasted for about
2 months. She has found it difficult to get up from a chair, and brushing her hair has become more problematic. In addition, she has developed blanching of the hands with exposure to cold, as well as stiffness of the hands, wrists, and feet lasting for 1 to 2 hours in the morning. She can no longer wear her wedding ring because of finger swelling. She also notes that she gets out of breath easily after climbing just one flight of stairs. On examination, she has evidence of proximal muscle weakness; synovitis of the MCPs, PIPs, wrists, and MTPs; and rales at the lung bases. There is no evidence of rash or lower extremity edema, and cardiac examination is normal. You suspect a connective tissue disease, and indeed, the screening ANA is positive at a titer of 1:640.

What additional antinuclear test will be most useful diagnostically for this patient?
A. Anti–Scl-70
B. Anti-dsDNA
C. Anti-SSA
D. Anti–Jo-1
E. Anti-Sm

Key Concept/Objective: To know the extramuscular manifestations of polymyositis associated with Jo-1 antibodies

Patients with polymyositis frequently have extramuscular manifestations. One of the most common is pulmonary fibrosis. Almost 70% of patients with pulmonary fibrosis will have the autoantibody Jo-1 in their serum. Anti–Jo-1 is one of the antisynthetase antibodies currently found only in patients with myositis. Besides pulmonary fibrosis, the antisynthetase syndrome includes Raynaud phenomenon, polyarthritis, and, in some cases, so-called mechanic’ s hands. The last condition causes cracked and fissured skin on the hands. AntidsDNA and anti-Sm are autoantibodies found only in systemic lupus erythematosus, antiScl-70 is seen in patients with scleroderma, and anti-SSA is found in patients with either systemic lupus erythematosus or Sjögren syndrome. (Answer: D—Anti–Jo-1)

31. A 50-year-old woman with a 6-month history of polymyositis complains of increasing weakness. After diagnosis, she was started on 60 mg of prednisone a day, which has been tapered to 30 mg/day. The initial CK was 3,000 mg/dl, but 3 weeks ago it was only slightly elevated to 400 mg/dl (normal, greater than or equal to 275 mg/dl). Examination demonstrates cushingoid facies, 4/5 proximal muscle strength, and no abnormal heart or lung findings. Current CK is 375 mg/dl.

Of the following, which is the best step to take next in the treatment of this patient?
A. Increase prednisone to 60 mg/day and reevaluate in 2 weeks
B. Refer to surgery for biopsy of one of the quadriceps muscles
C. Decrease prednisone to 20 mg/day and reevaluate in 2 weeks
D. Add methotrexate, 7.5 mg/wk
E. Refer to physical therapy to initiate strengthening exercises

Key Concept/Objective: To be able to recognize steroid myopathy

This patient with polymyositis has evidence of steroid myopathy. There is an increasing sense of proximal weakness without any increase in the CK. The best way to determine whether steroid myopathy is contributing to the weakness is to try a steroid taper and see if the weakness improves. If so, a second-line agent such as methotrexate would be useful, although even methotrexate may take several weeks to months to be effective. Biopsy of the muscle may show type 2 fiber atrophy typical of steroid myopathy, but in the setting of polymyositis, the diagnosis may be difficult to interpret. (Answer: C—Decrease prednisone to 20 mg/day and reevaluate in 2 weeks)

32. A 34-year-old woman complains of weakness, fatigue, hair loss, and numbness of the fingers. Her symptoms began 4 months ago, soon after the delivery of her second child. While visiting her mother, she saw her mother’ s physician for the above complaints and was found to have a CK of 600 mg/dl. She was told to see her local physician on returning home for evaluation of possible polymyositis. On examination, blood pressure is 90/60 mm Hg; pulse is 60 beats/min; hair appears thin; lungs and heart are normal; muscle strength is 5/5 in both the proximal and distal groups; and Phalen testing is positive at both wrists.

Of the following, which is the best test to perform next in the evaluation of this patient?
A. Electromyogram (EMG) study
B. Sensitive TSH
C. Aldolase
D. ANA panel
E. Repeat CK

Key Concept/Objective: To know that the differential diagnosis of polymyositis includes hypothyroidism

Hypothyroidism can cause all of the symptoms experienced by this patient as well as an elevated CK. CK levels generally do not reach those seen in inflammatory myositis. Rheumatologic manifestations of hypothyroidism include arthralgias and even joint swelling, myalgias and muscle cramps, carpal tunnel syndrome (which this patient has), and nonspecific paresthesias. (Answer: B—Sensitive TSH)

33. A 54-year-old woman with a recent diagnosis of dermatomyositis is referred to you for further evaluation. She has read that dermatomyositis can be associated with malignancy. She has recently had a thorough physical examination, chest x-ray, stool screening for occult blood, mammogram, and pelvic examination, all of which were deemed unremarkable.

What would you recommend to this patient?
A. Reassure her with regards to malignancy
B. Set up a routine-visit schedule for screening
C. Suggest that a CEA be checked today
D. Suggest that a CA-125 and transvaginal ultrasound be performed
E. Suggest that a colonoscopy be done soon

Key Concept/Objective: To understand the relationship between dermatomyositis and malignancy

It is generally accepted that patients with dermatomyositis are at increased risk of malignancy, the relative risk of which is approximately four to five times that of control groups. Cancers of the ovary, lung, lymphatic system, and hematopoietic system are overrepresented in patients with dermatomyositis. In women older than 40 years with dermatomyositis, the risk of ovarian cancer is 20 times that of the general population. Ovarian tumors are notoriously hard to find early. The most recent recommendations for detecting them in patients with dermatomyositis include a careful gynecologic examination, measurement of CA-125, and transvaginal ultrasound at 3to 6-month intervals. (Answer: D— Suggest that a CA-125 and transvaginal ultrasound be performed)

34. A 45-year-old man is referred to you for a workup of muscle weakness. He presented 2 months ago with proximal weakness and an elevated CK of 4,000 mg/dl. A recent biopsy showed no abnormalities. On examination, the patient has 4/5 strength in the proximal muscles and 5/5 strength distally; otherwise, the examination is normal.

Of the following, which is the best step to take next in the management of this patient?
A. Order an MRI of the quadriceps muscles
B. Order an EMG study of the proximal muscles
C. Refer the patient to a muscular dystrophy clinic
D. Repeat CK after patient has a period of rest
E. Perform the ischemic forearm test

Key Concept/Objective: To understand the use of MRI in improving the diagnostic accuracy of biopsy

Involvement of muscles in polymyositis is often patchy. In recent years, the use of MRI of the proximal muscles has demonstrated the patchy nature of the disease and aided in the localization of biopsy. This is the most likely reason for this patient’s normal biopsy. An MRI scan will probably demonstrate the extent and location of muscle disease, and rebiopsy of involved sites will most likely demonstrate myositis. (Answer: A—Order an MRI of quadriceps muscles)

For more information, see Olsen NJ, Brogan BL: 15 Rheumatology: VI Idiopathic Inflammatory Myopathies. ACP Medicine Online (www.acpmedicine.com). Dale DC, Federman DD, Eds. WebMD Inc., New York, January 2003

Systemic Vasculitis Syndromes

35. A 47-year-old woman presents to your clinic complaining of crampy abdominal pain and diarrhea; she also has been experiencing progressive dyspnea on exertion and constant chest pain that is worse when she leans forward. She has had these symptoms for the past several days. She also reports arthralgias and has now developed a rash. On physical examination, the patient is mildly tachypneic, but her vital signs are otherwise stable. Pulmonary examination reveals dullness to percussion and decreased breath sounds over the right lower lung field, with no egophony. Cardiac and abdominal examinations are unremarkable. Lower extremities are notable for purpura and trace edema below the knees. Chest x-ray reveals a right pleural effusion. Evaluation of her diarrhea reveals an eosinophilic gastroenteritis. You suspect that her underlying problem is Churg-Strauss syndrome (CSS).

Which of the following components of this patient’s medical history would be most supportive of the diagnosis of CSS?
A. A preceding streptococcal or viral infection
B. Chronic sinusitis
C. Bronchial asthma
D. Polymyalgia rheumatica

Key Concept/Objective: To understand the importance of history in the diagnosis of ChurgStrauss syndrome

CSS displays clinical similarities to Wegener granulomatosis (WG) in terms of organ involvement and pathology, especially in patients with upper or lower airway disease or glomerulonephritis. CSS can follow a rapidly progressive course. CSS differs most strikingly from WG in that the former occurs in patients with a history of atopy, asthma, or allergic rhinitis, which is often ongoing. In the prevasculitic atopy phase, as well as during the systemic phase of the illness, eosinophilia is characteristic and often of striking degree (= 1,000 eosinophils/mm3). When eosinophilia is present in WG, it is usually more modest (~500 eosinophils/mm3). Chronic sinusitis can be seen in both CSS and WG, although it is more characteristic in the latter than the former. Polymyalgia rheumatica is not associated with either CSS or WG; there is, however, a clear association between polymyalgia rheumatica and temporal arteritis. A preceding streptococcal or viral infection has been seen occasionally with both WG and CSS. (Answer: C—Bronchial asthma)

36. A 54-year-old man is brought to the emergency department by his family. They report that several days ago, the patient began complaining of arthralgias, myalgias, and subjective fevers. He thought that he had the flu and remained home from work. Yesterday he developed swelling and a rash on his legs. According to his family, yesterday evening the patient started acting funny, and today he has been somewhat confused. On physical examination, the patient’s temperature is 99.5° F (37.5° C); his heart rate is 93 beats/min, and his blood pressure is 154/85 mm Hg. He is able to answer questions but is easily distracted during the examination. Pulmonary, cardiovascular, and abdominal examinations are normal. On musculoskeletal examination, petechiae and purpura are noted on the upper and lower extremities, with 1+ pitting edema in the lower extremities. Laboratory values reveal a white blood cell count of 24,000, a platelet count of 550,000, and a hematocrit of 35%. Blood urea nitrogen and creatinine levels are 120 mg/dl and 4.5 mg/dl, respectively. You admit the patient to the hospital for further workup. Renal biopsy reveals pauci-immune glomerulonephritis. A serum test for perinuclear antineutrophil cytoplasmic antibodies (p-ANCAs) with antimyeloperoxidase specificity is positive.

Which of the following vasculitides is most likely responsible for this man’s illness?
A. Polyarteritis nodosa (PAN)
B. Allergic granulomatous angiitis
C. Microscopic polyangiitis (MPA)
D. Wegener granulomatosis (WG)

Key Concept/Objective: To know the clinical presentation and laboratory findings for MPA

Glomerulonephritis, particularly rapidly progressive glomerulonephritis, and alveolar hemorrhage are common in MPA and absent, by definition, in classic PAN. Constitutional symptoms such as fever, asthenia, and myalgias are common in both PAN and MPA. Elevated acute-phase reactants, thrombocytosis, leukocytosis, and the anemia of inflammatory disease are common, although they are not uniformly present. The diagnosis of MPA and PAN should ideally be based on histopathologic demonstration of arteritis and the clinical pattern of disease. A biopsy specimen of clinically involved, nonnecrotic tissue that demonstrates the presence of arteritis of muscular arteries is the ideal supportive finding for the diagnosis of arteritis of a medium-sized vessel, but such a finding is not always possible. The presence of serum p-ANCA with antimyeloperoxidase specificity (found in
60% of MPA patients) supports the clinical diagnosis of MPA, but p-ANCA is not specific for this disease. ANCAs are not characteristic of PAN. The renal biopsy tissue in MPA, as in WG and CSS, does not contain extensive immune complexes on immunofluorescent staining and electron microscopy (so-called pauci-immune glomerulonephritis). (Answer: C— Microscopic polyangiitis [MPA])

37. A 14-year-old girl is brought in to the emergency department by her parents. They report high spiking fevers that began several days ago. They also report that the patient has complained of headaches and aching joints and that today she developed a rash. On physical examination, the patient’s temperature is 103.2° F (39.5° C), her heart rate is 110 beats/min, and her blood pressure is 125/70 mm Hg. You note nonexudative conjunctivitis and an erythematous, dry oropharynx. Pulmonary and cardiac examinations are unremarkable except for tachycardia. Her distal limbs are notable for mild swelling. Skin examination reveals a diffuse, polymorphous rash with some plaques. You seriously suspect Kawasaki disease (KD) and begin administration of I.V. immunoglobulin and aspirin.

The morbidity and mortality of KD is associated with which of the following complications?
A. Overwhelming sepsis caused by encapsulated organisms
B. Central hypertension and stroke
C. Glomerulonephritis and renal failure
D. Coronary artery aneurysms and thrombosis

Key Concept/Objective: To know the life-threatening complications of KD

The morbidity and mortality (< 3%) of KD is overwhelmingly associated with the development of inflammatory coronary artery aneurysms, most of which are asymptomatic at the time of formation. Aneurysms may be detected by echocardiography. Thrombosis can occur in the aneurysms, resulting in direct or embolic coronary artery occlusion. Coronary events may occur weeks or even many years after the febrile illness. A baseline echocardiogram should be obtained at the time of the acute illness and should be repeated 2 and 6 weeks later. Early recognition of the disease and treatment with intravenous immunoglobulin and aspirin have significantly decreased the frequency of aneurysm formation and thrombotic coronary events. Renal compromise is distinctly unusual in KD. (Answer: D—Coronary artery aneurysms and thrombosis)

38. An 18-year-old woman comes to your clinic complaining of a rash on her legs. She reports having crampy abdominal pain and aching joints for several days. She also reports that the rash began yesterday evening and was worse this morning, and she complains that her skin is itchy. Her medical history is significant only for an upper respiratory infection 2 or 3 weeks ago that resolved spontaneously. She is otherwise healthy. Physical examination is notable only for trace edema and purpura, noted on both lower extremities. You suspect small vessel vasculitis. You perform a skin biopsy, which stains positively for IgA-containing immune complexes.

This biopsy finding is most consistent with which of the following diseases?
A. Henoch-Schönlein purpura
B. Urticarial vasculitis
C. CSS
D. WG

Key Concept/Objective: To know the clinical presentation and pathologic findings of HenochSchönlein purpura

Cutaneous involvement can occur in many of the primary or secondary vasculitic syndromes. Large, medium-sized, or small vessel occlusion can cause livedo, Raynaud phenomenon, or necrosis. Purpura is the most common manifestation of small vessel vasculitis. Small vessel vasculitis, particularly when associated with infections, is frequently associated with immune complex deposition. Vasculitis primarily involving the postcapillary venules has been termed hypersensitivity vasculitis in older literature. Primary small vessel vasculitis may be limited to the skin or may be associated with visceral involvement, including alveolar hemorrhage; intestinal ischemia or hemorrhage; and glomerulonephritis. Purpura tends to occur in recurrent crops of lesions of similar age and is more pronounced in gravity-dependent areas. Biopsy is useful in excluding causes of nonvasculitic purpura such as amyloidosis, leukemia cutis, Kaposi sarcoma, T cell lymphomas, and cholesterol or myxomatous emboli. Tissue immunofluorescent staining is useful to support the diagnosis of Henoch-Schönlein purpura (specifically, IgA staining), systemic lupus erythematosus, or infection (the percentage of cases with positive results on immunofluorescent staining is not known). Patients with WG and CSS can present with purpura; however, they do not exhibit IgA deposits in the immunoflourescence stains. Urticarial vasculitis is a disease that affects the skin exclusively; very rarely, patients present with interstitial lung disease but not articular or abdominal complaints, as seen in this patient. (Answer: A— Henoch-Schönlein purpura)

For more information, see Mandell BF: 15 Rheumatology: VIII Systemic Vasculitis Syndromes. ACP Medicine Online (www.acpmedicine.com). Dale DC, Federman DD, Eds. WebMD Inc., New York, August 2003

Crystal-Induced Joint Disease

39. A 67-year-old man comes to your clinic to establish care. He has a history of hypertension, gout, obesity, and hyperlipidemia. He denies alcohol abuse. He tells you that he has not had a “gout flare” in several years and takes no medicines for this condition. His medications include a dihydropyridine calcium channel blocker and a statin. He does not take aspirin. You order routine laboratory studies, including assessment of the uric acid level. All laboratory results are normal.

For this patient, which of the following statements regarding gout is false?

A. Hyperuricemia must be present to make a diagnosis of gout
B. This patient likely has secondary gout
C. Gout is primarily a disease of middle-aged men
D. Obesity, alcohol intake, high blood pressure, and an elevated serum creatinine level correlate with elevation of the serum uric acid level and the development of gout
E. In 80% to 90% of patients with primary gout, hyperuricemia is caused by underexcretion of uric acid in the presence of normal renal function

Key Concept/Objective: To understand that although hyperuricemia is associated with gout, it does not always lead to the development of gout

The development of gout tends to be associated with chronically increased levels of serum uric acid. However, a substantial minority of patients with acute gout will have normal uric acid levels, and hyperuricemia does not always lead to the development of gout. Gout is often classified as primary or secondary. Gout associated with an inborn problem in metabolism or decreased renal excretion without other renal disease is referred to as primary gout, whereas gout associated with an acquired disease or use of a drug is called secondary gout. In both primary and secondary gout, chronic hyperuricemia may be the result of overproduction of uric acid caused by increased purine intake, synthesis, or breakdown, or it may be the result of decreased renal excretion of urate. Gout is predominantly a disease of middle-aged men, but there is a gradually increasing prevalence in both men and women in older age groups. In most studies, the annual incidence of gout in men is one to three per 1,000; the incidence is much lower in women. Additional factors that correlate strongly with serum urate levels and the prevalence of gout in the general population include serum creatinine levels, body weight, height, blood pressure, and alcohol intake. Hyperuricemia can result from decreased renal excretion or increased production of uric acid. In 80% to 90% of patients with primary gout, hyperuricemia is caused by renal underexcretion of uric acid, even though renal function is otherwise normal. (Answer: A— Hyperuricemia must be present to make a diagnosis of gout)

40. A 74-year-old man presents to your clinic with a 2-day history of pain in his right great toe. You suspect gout and recommend treatment. You order laboratory studies, and the patient’s serum uric acid level is found to be elevated. Before leaving your office, the patient asks you what he should expect in the future concerning this disease.

In counseling this patient about the clinical presentation and course of this condition, which of the following statements is false?
A. Initial attack of gout is monoarticular in 85% to 90% of cases, and half of these cases will involve the first metatarsophalangeal joint
B. The presence of fever and the involvement of multiple joints effectively rules out the diagnosis of gout
C. If the patient’s hypouricemia is not treated, there is at least a 75% chance of further attacks within 2 years and a 90% chance within 10 years
D. Without treatment for hyperuricemia, the patient will likely develop tophi within 12 years
E. Chronic gout eventually progresses to articular destruction, including bony erosions that may have the radiographic appearance of punchedout lesions of periarticular bone, often with an overhanging edge that can be distinguished from rheumatoid arthritis

Key Concept/Objective: To understand that gout may involve multiple joints and be associated with fever

Acute gouty arthritis is usually characterized by a sudden and dramatic onset of pain and swelling, usually in a single joint. This condition occurs most often in lower extremity joints and evolves within hours to marked swelling, warmth, and tenderness. The initial attack of gout is monoarticular in 85% to 90% of patients. At least half of initial attacks occur in the first metatarsophalangeal joints (a condition known as podagra), but other joints of the foot may be involved simultaneously or in subsequent attacks. Other lower extremity joints, including the ankles and knees, are often affected; in more advanced gout, attacks may occur in upper extremity joints, such as the elbow, wrist, and small joints of the fingers. Polyarticular gout occurs as the initial manifestation in about 10% to
15% of patients and may be associated with fever. After the initial attack of gout subsides, the clinical course may follow one of several patterns. A minority of patients never have another attack of gout, and some may not have another attack for several years. Most patients, however, have recurrent attacks over the ensuing years. In a study conducted before the use of hypouricemic agents, 78% of patients had a second attack within 2 years and 93% had a second attack within 10 years. Persistent hyperuricemia with increasingly frequent attacks of gout eventually leads to joint involvement of wider distribution, as well as chronic joint destruction as a result of deposition of massive amounts of urate in and around joints. Without therapy to lower serum uric acid levels, the average interval from the first gouty attack to the development of chronic arthritis or tophi is about 12 years. Erosive bony lesions may be seen on x-rays as well-defined, punched-out lesions in periarticular bone, often associated with overhanging edges of bone. These erosions are usually
5 mm or more in diameter and are larger than those seen in rheumatoid arthritis. Bone mineralization appears to be generally normal in chronic tophaceous gout, and periarticular osteopenia, which is seen in rheumatoid arthritis, is usually not present. The distribution of destructive joint disease in gout is often asymmetrical and patchy. (Answer: B— The presence of fever and the involvement of multiple joints effectively rules out the diagnosis of gout)

41. A 77-year-old male patient of yours presents to your clinic for evaluation of right knee pain. He has had recurrent gout in this knee, and his current symptoms are consistent with previous presentations. This patient has many medical problems, which include congestive heart failure, chronic kidney disease, and hypertension. Three months ago, the patient was admitted to the hospital for upper gastrointestinal bleeding; he was found to have peptic ulcer disease. His medications include a daily dose of colchicine, but he admits that he has not taken his medications today because of the pain and mild nausea. The patient is noticeably uncomfortable. His examination is notable only for marked swelling and erythema of the right knee and the presence of an effusion.

Which of the following treatment strategies should be prescribed for this patient’s gouty attack?
A. Nonsteroidal anti-inflammatory drugs (NSAIDs), such as indomethacin
B. Colchicine, 0.6 mg to 1.2 mg initially and then 0.6 mg every 2 hours until the flare resolves
C. Allopurinol
D. Cyclooxygenase-2 inhibitors
E. Arthrocentesis followed by administration of an intra-articular steroid to provide immediate relief

Key Concept/Objective: To understand the management of acute gouty attacks in patients with multiple comorbidities

Treatment of acute gout should be initiated as early in the attack as possible. Agents available for terminating the acute attack include colchicine, NSAIDs, adrenocorticotropic hormone (ACTH), and corticosteroids. Each agent has a toxicity profile, with advantages and disadvantages applicable to individual circumstances. This patient’s overall health and coexistent medical problems, particularly renal and gastrointestinal disease, dictate the choice among these approaches. Corticosteroids and ACTH have been used more often in recent years in patients with multiple comorbid conditions, because of the relatively low toxicity profile of these agents. Colchicine has been used for centuries to treat acute attacks of gout. Given in oral dosages of 0.6 to 1.2 mg initially, followed by 0.6 mg every 2 hours, colchicine begins relieving most attacks of gout within 12 to 24 hours. However, most patients experience nausea, vomiting, abdominal cramps, and diarrhea with these dosages. Colchicine should be given more cautiously in elderly patients and should be avoided in patients with renal or hepatic insufficiency and in patients who are already receiving long-term colchicine therapy. NSAIDs are useful in most patients with acute gout and remain the agents of choice for young, healthy patients without comorbid diseases. The use of all NSAIDs is limited by the risks of gastric ulceration and gastritis, acute renal failure, fluid retention, interference with antihypertensive therapy, and, in older patients, problems with mentation. Cyclooxygenase-2–specific NSAIDs should be useful in treating acute gout and possibly for long-term prophylaxis in patients at risk for gastrointestinal toxicity from the currently available NSAIDs but are not without risk in patients with renal insufficiency and congestive heart failure. The use of intra-articular steroids after arthrocentesis is extremely useful in providing relief, particularly in large effusions, in which the initial aspiration of fluid results in rapid relief of pain and tightness in the affected joint. The dosage of the steroid triamcinolone depends on the size of the joint; dosages range from 5 to 10 mg for small joints of the hands or feet to 40 to 60 mg for larger joints, such as the knee. Systemic corticosteroids may also be useful in patients for whom colchicine or NSAIDs are inadvisable and for patients with polyarticular attacks. (Answer: E—Arthrocentesis followed by administration of an intra-articular steroid to provide immediate relief)

42. A 55-year-old man presents with a painful swollen right great toe. He reports a previous similar attack 3 months ago. The pain is severe, even with minimal pressure from his sock or bed sheet. The medical history includes reflux esophagitis, GI bleeding, and COPD. Laboratory results include the following: uric acid, 8.8; Ca, 9.3; Na, 144; K, 5.0; BUN, 26; Cr, 2.9; and Glu, 96.

What is the best treatment option for this patient?
A. Indomethacin, 50 mg p.o., t.i.d
B. Prednisone taper
C. Rofecoxib, 25 mg p.o., q.d.
D. Allopurinol, 300 mg p.o., q.d.
E. Acetaminophen, 1 g p.o., q.i.d.

Key Concept/Objective: To appreciate comorbid conditions when selecting treatment for acute gout

This patient’s acutely painful great toe is suggestive of gout. His uric acid level is high, which is consistent with this diagnosis. Appropriate treatment would be either oral prednisone or steroids injected into the joint. He should not receive NSAIDs because he has renal insufficiency. Rofecoxib, a COX-2 inhibitor, can also be detrimental to renal function and should not be used in this setting. Allopurinol is not indicated for the acute treatment of gout. (Answer: B—Prednisone taper)

43. A 54-year-old man presents with an attack of gout. He had his first attack 6 months ago; gout was confirmed by joint aspiration that revealed uric acid crystals. Three months later, he had a second attack, which involved his knee and left great toe. The medical history includes hypertension, reflux esophagitis, and psoriasis. The patient reports drinking one glass of wine once a week. Medications include omeprazole, lisinopril, hydrochlorothiazide, and triamcinolone ointment.

What would you recommend for this patient to prevent future episodes of gout?
A. Stop all alcohol use
B. Stop hydrochlorothiazide
C. Begin allopurinol
D. Begin probenecid
E. Begin colchicine

Key Concept/Objective: To understand that hydrochlorothiazide is a common trigger for elevated uric acid and gout

This patient has had several attacks of gout over a 6-month period. Hydrochlorothiazide can decrease uric acid excretion and raise uric acid levels, triggering attacks of gout. Before considering use of prophylactic medications in this patient, it would be appropriate to withhold the hydrochlorothiazide and see whether the gout attacks stop. Alcohol consumption can also precipitate attacks, but this patient’s infrequent alcohol use is unlikely to be the cause of his gout attacks. (Answer: B—Stop hydrochlorothiazide)

44. A 51-year-old man presents for primary care. He has no major medical problems. He brings with him old records that include results of lab testing done a year ago. Those results were as follows: Na, 139; K, 4.2; Cl, 100; HCO3, 26; BUN, 12; Cr, 1.0; uric acid, 10.2. Laboratory tests are repeated, and the results are normal, with the exception of a uric acid measurement of 10.4.

What would you recommend for this patient?
A. No therapy
B. Allopurinol
C. Probenecid
D. Colchicine
E. Aspirin

Key Concept/Objective: To understand that asymptomatic hyperuricemia does not need therapy

This patient has asymptomatic hyperuricemia. There is no need to treat asymptomatic patients with hypouricemic agents. They should be followed closely for the development of gout or renal stones. If either condition develops, it would be appropriate to consider treatment. (Answer: A—No therapy)

45. A 61-year-old man presents with a swollen, warm, tender left knee. He has had three episodes of gout this year, which were treated successfully with indomethacin. He is started on a 1-week course of indomethacin.

What other therapy would be appropriate to start?
A. Prednisone, 40 mg q.d. for 3 days
B. Probenecid, 1 g p.o., b.i.d.
C. Colchicine, 0.6 mg p.o., b.i.d.
D. Allopurinol, 300 mg p.o., q.d.
E. Acetaminophen, 1 g p.o., b.i.d.

Key Concept/Objective: To understand how and when to start prophylactic medications for gout

This patient presents with an acute attack of gout. He has had several episodes in the past year. It would be appropriate to treat him with indomethacin for the acute attack and to begin medication to decrease the risk of another attack in the near future. Colchicine, 0.6 mg once or twice a day, prevents recurrent attacks in 80% of patients with gout. It should be started in conjunction with acute treatment (NSAIDs or steroids) and continued for 1 to 2 months. Colchicine should also be used when urate-lowering drug therapy is initiated. This patient should not receive allopurinol or probenecid during the acute attack because both of these agents can worsen the acute attack. There is no need to add prednisone to the indomethacin being used for acute treatment. (Answer: C—Colchicine, 0.6 mg p.o., b.i.d.)

For more information, see Wise C: 15 Rheumatology: IX Crystal-Induced Joint Disease. ACP Medicine Online (www.acpmedicine.com). Dale DC, Federman DD, Eds. WebMD Inc., New York, March 2004

Osteoarthritis

46. A 51-year-old white woman presents to your primary care clinic for evaluation of knee pain. She states that the pain has been progressing gradually for at least a year. The patient denies having had any trauma. She also states that she has not experienced any erythema, point tenderness, fevers, or chills, nor has she lost the ability to ambulate. However, she occasionally notes some swelling of the joint. Physical examination is notable for the absence of joint instability, fever, redness, edema, or warmth. You suspect that the patient has osteoarthritis.

Which of the following statements regarding osteoarthritis is false?
A. In patients older than 50 years, men are more commonly affected than women
B. In most patients with primary osteoarthritis, involvement is limited to one or a small number of joints or joint areas
C. Intra-articular fractures and meniscal tears can lead to osteoarthritis years after the injury
D. Most patients with radiographic changes consistent with osteoarthritis have few symptoms or functional limitations

Key Concept/Objective: To understand the epidemiology and etiology of osteoarthritis

Osteoarthritis is a common form of arthritis characterized by degeneration of articular cartilage and reactive changes in surrounding bone and periarticular tissue. The disease process results in pain and dysfunction of affected joints and is a major cause of disability in the general population. Patients without a specific inflammatory or metabolic condition known to be associated with arthritis who have a history of specific injury or trauma are considered to have primary osteoarthritis. In most patients, involvement is limited to one or a small number of joints or joint areas. Secondary osteoarthritis has been associated with several conditions that cause damage to articular cartilage through a variety of mechanisms, including mechanical, inflammatory, and metabolic processes. Acute trauma, particularly intra-articular fractures and meniscal tears, can result in articular instability or incongruity and can lead to osteoarthritis years after an injury. Osteoarthritis is the most common type of arthritis, and it is one of the most common causes of disability and dependence in the United States. Fortunately, most patients with radiographic changes found in population-based surveys have few symptoms or functional limitations. Men and women tend be affected equally by osteoarthritis in middle age, but after the age of 50 years, women are more commonly affected than men. (Answer: A—In patients older than
50 years, men are more commonly affected than women)

47. A 32-year-old man presents to your clinic for management of hypertension. He states that his mother and father and many of their siblings have arthritis, and he wants to know if he will get arthritis, too. He has no current complaints of arthritis or arthralgias, and his joint examination is normal.

Which of the following statements regarding risk factors for osteoarthritis is false?
A. Age and presence of osteoarthritis are positively correlated
B. Despite many well-designed studies, an association between obesity and osteoarthritis has not been established
C. Bone mineral density and the presence of osteoarthritis are positively correlated
D. A family history of osteoarthritis is common in patients with osteoarthritis

Key Concept/Objective: To understand the risk factors for osteoarthritis

A number of risk factors are believed to contribute to the development of primary osteoarthritis, including age, obesity, joint malalignment, bone density, hormonal status, nutritional factors, joint dysplasia, trauma, occupational factors, and hereditary factors. Age is the factor most strongly associated with radiographic and clinically significant osteoarthritis, with an exponential increase seen in more severely involved joints. Obesity is clearly associated with osteoarthritis of the knee. The increased load carried by obese persons and the alterations in gait and posture that redistribute the load contribute to cartilage damage. A study in young men suggested that each increase in weight of 8 kg results in a 70% increase in the risk of symptomatic arthritis of the knee in later years. This association is particularly high in patients with varus malalignment of the knee, and obese patients with malalignment are at risk for more rapid progression of established osteoarthritis in the knee. Most of the association of obesity with osteoarthritis of the knee appears to be related to environmental, rather than genetic, factors. An association between increased bone density and osteoarthritis has been noted in several studies. Women with osteoporosis and hip fractures have a decreased risk of osteoarthritis, and those affected by osteoarthritis have significantly increased bone density. This negative association suggests that soft subchondral bone absorbs impact and protects articular cartilage better than dense bone. Many patients with osteoarthritis have a family history of the disorder, and multiple genetic factors may be responsible in various forms of osteoarthritis. In women, osteoarthritis with finger joint involvement is probably the bestrecognized form of arthritis with familial associations, but hereditary factors are also important in osteoarthritis of the hip. (Answer: B—Despite many well-designed studies, an association between obesity and osteoarthritis has not been established)

48. A 64-year-old white woman presents to the clinic with a 3to 4-month history of worsening right hand pain. She denies undergoing any trauma or injury, and she states that her pain is worse at the base of the thumb. On physical examination, the patient has bony enlargement of her distal and proximal interphalangeal joints. The carpometacarpal joint of the right thumb is exquisitely painful to motion, but there is no overlying erythema or edema.

Which of the following statements regarding the clinical manifestations and diagnostic tests for osteoarthritis is true?
A. The most commonly involved joints are the wrists, the metacarpophalangeal joints, the elbows, the shoulders, and the ankles
B. The ESR is usually elevated, and it is common to find an elevated leukocyte count (> 2,000 cells/mm3) in the synovial fluid
C. Synovial effusions may be present; erythema and warmth suggest the presence of coexistent crystal-induced inflammation or other conditions
D. Morning stiffness can occur with osteoarthritis and typically will last longer than 1 hour

Key Concept/Objective: To understand the clinical manifestations of osteoarthritis and the diagnostic tests used in the workup

Typical symptoms of osteoarthritis include pain, stiffness, swelling, deformity, and loss of function. Pain is usually chronic and localized to the involved joint or joints or referred to nearby areas. Pain may be mild or moderate early in the disease but tends to worsen gradually over many years. Most of the pain is made worse with activity and improves with rest. Morning stiffness is not as prolonged as it is in patients with inflammatory diseases; morning stiffness in patients with osteoarthritis usually lasts less than an hour. Physical findings in osteoarthritis include crepitus, pain on motion, bony enlargement, and periarticular tenderness. Synovial effusions may be present, particularly in the knee. Erythema and warmth are unusual and should suggest the presence of coexistent crystal-induced inflammation or other conditions. Osteoarthritis has a characteristic pattern of involvement in most patients. Frequently involved joints include the distal and proximal interphalangeal joints and the first carpometacarpal joints in the hands; the cervical and lumbar spine; the hips; the knees; and, less commonly, the small joints of the feet or the acromioclavicular joint. The wrists, metacarpophalangeal joints, elbows, shoulders, and ankles are usually not affected unless there is a history of injury to the specific joint, occupational overuse, or an underlying condition that might be a cause of secondary osteoarthritis. Characteristic radiographic features are usually considered essential for diagnosis but should be corroborated by the presence of compatible symptoms. Laboratory studies are useful in the evaluation of patients with osteoarthritis only in that they help to exclude other diagnoses. Thus, the ESR, rheumatoid factor, and routine hematologic and biochemical parameters should be normal in patients with osteoarthritis unless the osteoarthritis is attributable to comorbid conditions. Synovial fluid from involved joints is noninflammatory, with leukocyte counts of less than 2,000 cells/mm3 in most patients. Typical radiographic findings in osteoarthritis include joint space narrowing, subchondral bone sclerosis, subchondral cysts, and osteophytes (bony spurs). (Answer: C—Synovial effusions may be present; erythema and warmth suggest the presence of coexistent crystal-induced inflammation or other conditions)

49. A 46-year-old woman presents with complaint of pain, stiffness, and swelling in her right hand; these symptoms have persisted for several months. She denies experiencing any past or recent trauma to her hand or having any other significant medical history. On examination, she has tenderness on several of her distal and proximal interphalangeal joints and a Heberden node on the index finger.

Which of the following is the most likely diagnosis?
A. Primary osteoarthritis
B. Secondary osteoarthritis
C. Erosive osteoarthritis
D. Rheumatoid arthritis

Key Concept/Objective: To understand the classification of various forms of arthritis

Erosive osteoarthritis is characterized by polyarticular involvement of the small joints of the hand and tends to occur more often in middle-aged and elderly women. Patients without a specific inflammatory or metabolic condition known to be associated with arthritis and without a history of specific injury or trauma are considered to have primary osteoarthritis. Secondary osteoarthritis has been associated with several conditions that cause damage to articular cartilage through a variety of mechanisms, including mechanical, inflammatory, and metabolic processes. Rheumatoid arthritis can usually be distinguished from osteoarthritis on the basis of a different pattern of joint disease, more prominent morning stiffness, and soft tissue swelling and warmth on physical examination. (Answer: C—Erosive osteoarthritis)

50. A 50-year-old man presents with complaints of right knee pain and swelling of 4 days’ duration. He reports no new injury, but several years ago he underwent arthroscopic surgery in that knee for a meniscal tear. Since the time of his surgery, he has experienced intermittent pain in his knee when he “overdoes it,” but he has not previously experienced swelling in his knee. On examination, there is moderate effusion in the patient’s right knee, and range-of-motion assessment elicits crepitus and pain.

Which of the following may be found on radiographic examination of this patient’s right knee?
A. Joint space narrowing
B. Subchondral bone sclerosis
C. Osteophytes
D. All of the above

Key Concept/Objective: To understand the common radiographic findings of osteoarthritis

Typical radiographic findings in osteoarthritis include joint space narrowing, subchondral bone sclerosis, subchondral cysts, and osteophytes (bony spurs). Joint space narrowing, resulting from loss of cartilage, is often asymmetrical and may be the only finding early in the disease process. In weight-bearing joints such as the knees, narrowing may be seen only in a standing view and may be missed in a film obtained in the recumbent position. In more chronic disease, the hypertrophic features of subchondral sclerosis and osteophyte formation become more prominent, and subluxations or fusion of the joint may become apparent in more severely affected joints. In the small interphalangeal joints of the fingers, central erosions may be seen within the joint space. (Answer: D—All of the above)

51. A 33-year-old morbidly obese man presents for a routine physical examination. He reports pain in his knees, which he has been experiencing for several months and for which he takes acetaminophen. He denies undergoing any trauma to either knee. He also denies having any other past or present medical problems. On examination, both knees have crepitus with range-of-motion assessment, and the right knee has a small effusion.

Which of the following statements regarding this patient is false?
A. This patient has an increased risk of osteoarthritis of the knees
B. This patient should be counseled regarding dietary vitamin C and D supplementation
C. Analysis of the synovial fluid would show an absence of inflammation, with leukocyte counts below 2,000 cells/mm3
D. This patient would be expected to have an elevated erythrocyte sedimentation rate (ESR)

Key Concept/Objective: To understand the risk factors for and characteristics of nonpharmacologic measures for osteoarthritis

The ESR, rheumatoid factor level, and routine hematologic and biochemical parameters should be normal in patients with osteoarthritis unless the osteoarthritis is attributable to comorbid conditions. Laboratory studies are useful in the evaluation of patients with osteoarthritis only in that they help to exclude other diagnoses. Synovial fluid from involved joints is noninflammatory, with leukocyte counts being under 2,000 cells/mm3 in most patients. A number of risk factors are believed to contribute to the development of primary osteoarthritis, including age, obesity, bone density, hormonal status, nutritional factors, joint dysplasia, trauma, occupational factors, and hereditary factors. Obesity is clearly associated with osteoarthritis of the knee. The increased load carried by obese patients and the alterations in gait and posture that redistribute the load contribute to cartilage damage. Nonpharmacologic measures that have the potential to improve outcomes in osteoarthritis include patient education, physical and occupational therapy assessment and interventions, exercise, weight loss, and dietary vitamin D and C supplementation. Epidemiologic studies have suggested a role for adequate dietary vitamin C and D intake in reducing the risk of progression of established osteoarthritis. (Answer: D—This patient would be expected to have an elevated erythrocyte sedimentation rate [ESR])

52. A 67-year-old woman presents with pain and stiffness in various joints of her hands; these symptoms have persisted for several months. She denies experiencing any trauma to her hands. She also denies having any other relevant medical history. She states that she takes aspirin when she has pain. On examination, she has swelling on several proximal interphalangeal joints in both hands and has Bouchard nodes in two joints.

Which of the following is the most appropriate first-line pharmacologic treatment for this patient?
A. Acetaminophen
B. Opioids
C. Steroids
D. Cyclooxygenase-2 (COX-2) NSAIDs

Key Concept/Objective: To understand the pharmacologic therapies for osteoarthritis

Acetaminophen in doses up to 3,000 to 4,000 mg daily should be prescribed initially in most patients with osteoarthritis. The primary goal of drug therapy in osteoarthritis is to relieve pain. In some patients, simple analgesics may be as effective as NSAIDs. Opioids are generally avoided in osteoarthritis but may be useful in selected patients. Opioids should be used with caution in elderly patients. Tramadol, a centrally acting analgesic with dual mechanisms, may give relief comparable to that achieved with acetaminophen and codeine. Topical capsaicin may be useful in some patients, particularly those with involvement of the knees and hands. NSAIDs are useful in osteoarthritis mostly for their analgesic effects, although anti-inflammatory effects may have some clinical significance. NSAIDs are associated with an increased risk of gastric ulcers and bleeding, particularly in patients with a history of GI disease. The recently available COX-2–specific NSAIDs celecoxib and rofecoxib have been shown to reduce endoscopic gastritis and ulcers as well as serious GI complications when compared to the previously available nonselective COX inhibitors.
(Answer: A—Acetaminophen)

For more information, see Wise C: 15 Rheumatology: X Osteoarthritis. ACP Medicine Online (www.acpmedicine.com). Dale DC, Federman DD, Eds. WebMD Inc., New York, January 2005

Back Pain and Common Musculoskeletal Problems

53. A 67-year-old African-American man comes to your office with low back pain. He has been experiencing progressive back pain for the past 2 weeks. He believes these symptoms started after he lifted a 20-lb box. His medical history is unremarkable. Review of systems is significant for a weight loss of 10 lb over the past 6 months and urinary hesitancy. Physical examination reveals tenderness to percussion over L5 and a nodular, enlarged prostate. The neurologic examination is normal.

Of the following, which is the most appropriate step to take next in the treatment of this patient?
A. Start nonsteroidal anti-inflammatory drugs (NSAIDs) and have the patient come back to your clinic only if the pain persists
B. Prescribe bed rest for 1 week
C. Obtain imaging studies
D. Start opiates and muscle relaxants

Key Concept/Objective: To be able to identify patients with acute back pain who are at risk for serious underlying conditions

For patients with acute back pain, the initial history should be used to identify those who are at risk for serious underlying conditions, such as fracture, infection, tumor, or major neurologic deficit. The initial physical examination should include evaluation for areas of localized bony tenderness and assessment of flexion and straight leg raising. This patient has symptoms and signs that suggest the presence of a malignancy. He has experienced weight loss, and there is bony tenderness and a nodular prostate. In this clinical scenario, imaging is indicated to evaluate for the possibility of metastatic disease to the spine. For the treatment of acute back pain, NSAIDs and mild analgesics may be useful for symptom control. Muscle relaxants and opiates should be used sparingly. Spinal manipulation or specific exercise programs may also be effective in acute back pain. Over 90% of patients will improve within 1 month. Strict bed rest should be kept to a minimum, and continuation of normal activities should be enforced. (Answer: C—Obtain imaging studies)

54. A 42-year-old male postal worker presents to your clinic asking for a second opinion regarding the management of his chronic low back pain. The pain started 4 months ago. The pain is located in his lower back; it does not radiate. The patient denies having any weakness or sensory deficits. The pain is worse when he walks or when he lifts weights, and it is interfering with his work. The patient’s medical history and review of systems are unremarkable. He has tried over-the-counter acetaminophen and ibuprofen, without relief. Recently, he saw another physician, who ordered a magnetic resonance imaging scan. The report describes a bulging disk on L4-5 with no signs of spinal cord compression. On the basis of that study, the patient was told he needed surgery. On physical examination, there is diffuse tenderness in his lower back, and a leg-raising test is negative. The neurologic examination, including sphincter tone, is normal.

How would you manage this patient?
A. Prescribe an NSAID at a higher dosage than previously used, educate the patient about low back pain, and recommend physical therapy
B. Order a repeat MRI because the results do not fit with the physical examination
C. Refer to a neurosurgeon for surgical repair of his herniated disk
D. Recommend that the patient apply for disability because of his chronic pain

Key Concept/Objective: To understand the management of chronic back pain

A herniated lumbar disk should be considered in patients with back pain who have symptoms of radiculopathy, as suggested by pain radiating down the leg with symptoms reproduced by straight leg raising. MRI may be necessary to confirm a herniated disk, but findings should be interpreted with caution, because many asymptomatic persons have disk abnormalities. This patient has no signs of radiculopathy. Also, the MRI reports a bulging disk with no signs of compression: a finding that is frequently seen in healthy persons. Surgery would be indicated if there were signs of radiculopathy and the MRI showed a herniated disk with evidence of spinal compression; however, this is not the situation in this case. A repeat MRI is not indicated, because it is unlikely that a herniated disk is the cause of this patient’s symptoms, given the clinical evidence. The management of chronic back pain is complex. Patients should undergo physical therapy, an exercise program, and an education program that emphasizes proper ergonomics for lifting and other activities. Light normal activity and a regular walking program should be encouraged. Encouraging the patient to apply for disability before trying different therapeutic interventions is not appropriate. Judicious use of NSAIDs and mild analgesics may improve patient function and outcome. (Answer: A—Prescribe an NSAID at a higher dosage than previously used, educate the patient about low back pain, and recommend physical therapy)

55. A 55-year-old woman with a history of rheumatoid arthritis presents to the emergency department complaining of right elbow pain. The pain started 4 days ago and has become progressively worse, to the point where it is now difficult for her to move her elbow. She has also felt febrile. On physical examination, the patient’s temperature is 98.8° F (37.1° C), there are signs of chronic rheumatoid arthritis on her hands, and there is a 3 × 3 cm area of indurated swelling over the tip of her elbow. This area is tender to palpation and is warm and erythematous. The passive range of motion of the elbow is preserved.

What is the appropriate step to take next in the treatment of this patient?

A. Start NSAIDs and follow up within a week
B. Aspirate the fluid to rule out infection or crystal-induced disease
C. Order an MRI to evaluate the degree of joint damage
D. Inject steroids to the area

Key Concept/Objective: To be able to recognize different causes of olecranon bursitis

Olecranon bursitis presents as a discrete swelling with palpable fluid over the tip of the elbow. Olecranon bursitis may be secondary to trauma, rheumatoid arthritis, crystalinduced disease (e.g., gout or pseudogout), or infection. This patient’s clinical presentation should raise concern about an infectious process. She may have had a fever previously. On examination, she has an indurated, tender, erythematous area over her elbow. The ability to perform passive range of motion of the elbow makes the possibility of synovial infection unlikely; however, aspiration of the bursae is indicated to rule out infection and crystal-induced disease. Infectious bursitis, usually caused by gram-positive skin organisms, is accompanied by heat, erythema, and induration. When infection is suspected, prompt aspiration and culture of the fluid are mandatory. Antibiotics should be started empirically, and the bursae should be reaspirated frequently until the fluid no longer reaccumulates and cultures are negative. NSAIDs and steroids should not be started until the fluid has been examined, because of the risk of underlying infection. MRI is not indicated at this point, because the clinical picture is consistent with bursitis. (Answer: B—Aspirate the fluid to rule out infection or crystal-induced disease)

56. A 31-year-old obese man presents to your clinic with a 2-week history of right foot pain. The pain is located on his posterior heel. There is no history of trauma. The pain is worse when standing in the morning and when walking after sitting down for a period. On physical examination, there is tenderness to palpation on the heel area. The rest of the physical examination is normal.

Which of the following is the most likely cause of this patient’s symptoms

A. Previous unrecognized traumatic event
B. A deformity of the arch of his foot
C. Peripheral neuropathy
D. Plantar fasciitis

Key Concept/Objective: To know the causes of hindfoot pain

Plantar fasciitis is one of the most common causes of hindfoot pain. Patients report pain over the plantar aspect of the heel and midfoot that worsens with walking. Localized tenderness along the plantar fascia or at the insertion of the calcaneus is helpful in diagnosis. Plantar fasciitis is associated with obesity, pes planus, and activities that stress the plantar fascia. It may also be seen in systemic arthropathies such as ankylosing spondylitis and Reiter syndrome. Although radiographic spurs in the affected area are common, they may also be seen in asymptomatic persons and are therefore not diagnostic. In this case, the constellation of symptoms, obesity, and physical examination findings are consistent with the diagnosis of plantar fasciitis. Careful examination usually helps distinguish between Achilles tendinitis and plantar fasciitis. Therapy for plantar fasciitis is usually conservative and includes the use of orthotic devices (heel wedges), stretching exercises, and judicious use of NSAIDs. Heel spurs can be seen on plain radiography; however, their role in the pathogenesis of plantar fasciitis is unclear. (Answer: D—Plantar fasciitis)

For more information, see Wise C: 15 Rheumatology: XII Back Pain and Common Musculoskeletal Problems. ACP Medicine Online (www.acpmedicine.com). Dale DC, Federman DD, Eds. WebMD Inc., New York, December 2002

Fibromyalgia

57. A 34-year-old woman returns to your office for a routine follow-up visit. You diagnosed her as having fibromyalgia 3 years ago, when she presented with multiple tender points of muscle and tendons, marked sleep disturbance, recurrent headaches, fatigue, and chronic generalized pain. You have also treated her for generalized anxiety disorder and depression. Today, she states that her generalized pain is slightly improved. She is sleeping better, and her energy level has improved. She remembers that just before her pain syndrome started 3 years ago, she fell down her neighbor’s doorsteps. She asks you if you agree that this fall is the likely cause of her current pain syndrome.

Which of the following statements regarding fibromyalgia is true?
A. Fibromyalgia most commonly occurs in middle-aged men
B. Fibromyalgia is considered to be a purely somatic disease; social or psychological factors have little bearing on the disease
C. The type of pain associated with fibromyalgia is typically nociceptive or neuropathic
D. Fibromyalgia patients often have fixed beliefs that minor traumatic events or exposure to pathogens, chemicals, or other physical agents caused their illness

Key Concept/Objective: To know the general features of fibromyalgia

Fibromyalgia is a chronic syndrome that occurs predominantly in women. It is marked by generalized pain, multiple defined tender points, fatigue, disturbed or nonrestorative sleep, and numerous other somatic complaints. Fibromyalgia becomes more common after 60 years of age but also occurs in children. The cause of fibromyalgia is unknown. Despite extensive research, no definitive organic pathology has been identified. Psychological factors associated with chronic distress appear to be very important. In fibromyalgia, negative psychological elements constituting stress and distress are major contributors to the development of increased pain sensitivity and myriad other symptoms. There are four principal categories of pain: nociceptive, neuropathic, psychogenic, and chronic pain of complex etiology. Chronic pain of complex etiology is the type of pain characteristic of fibromyalgia. Fibromyalgia patients often have fixed beliefs that minor traumatic events, pathogens, chemicals, or other physical agents caused their illness. (Answer: D—Fibromyalgia patients often have fixed beliefs that minor traumatic events or exposure to pathogens, chemicals, or other physical agents caused their illness)

58. A 27-year-old woman visits your clinic as a new patient. She was in very good health until 1 year ago, when she developed severe neck, shoulder, and hip pain. Her primary physician has completed an extensive workup for rheumatologic disorders; the patient has brought the data from that workup with her today. The patient is in constant pain and has difficulty sleeping; she also has a “nervous stomach” and chronic diarrhea, and she feels that her “memory is slipping.” Her pains are so constant and severe that she has had to resign her job as a schoolteacher. Her social history reveals that she was divorced 1 year ago and is a single parent of three children.

Which of the following statements regarding the historical diagnosis of fibromyalgia is true?
A. Cognitive complaints, such as difficulty with concentration and memory, are notably absent in patients with fibromyalgia
B. Fibromyalgia does not lead to functional impairment
C. Regional pain syndromes, such as headache, temporomandibular joint syndrome, or irritable bowel syndrome, are uncommon in fibromyalgia
D. Pain is the hallmark of fibromyalgia

Key Concept/Objective: To understand important historical elements in patients with fibromyalgia

Cognitive complaints, such as difficulties with concentration and memory, may be prominent in fibromyalgia. Functional impairment is usually present, at least in patients with fibromyalgia who seek care. Patients report difficulty performing usual activities of daily living; in addition, they avoid exercise—indeed, patients with fibromyalgia are fearful of exercise. Regional pain syndromes, such as headache, temporomandibular joint syndrome, or irritable bowel syndrome, are extremely common in fibromyalgia. It is essential that the physician not automatically attribute all such symptoms to fibromyalgia, however, because fibromyalgia frequently coexists with other organically defined disorders. Pain is the hallmark of fibromyalgia. The pain radiates diffusely from the axial skeleton and is localized to muscles and muscle-tendon junctions of the neck, shoulders, hips, and extremities. (Answer: D—Pain is the hallmark of fibromyalgia)

59. A 35-year-old woman presents to your office with the complaints of severe joint pain, joint swelling, muscle aches, insomnia, and severe fatigue. All of her symptoms started 3 months ago when she lost her job as an executive assistant. She denies having fever or chills, unprotected sexual contact, morning stiffness, or gastrointestinal or urinary symptoms. On physical examination, diffuse swelling of the patient’s metacarpophalangeal joints and wrists is noted. The patient has an erythematous rash on her face. She has significant pain at 12 of the 18 tender points, and there is a mild reduction in strength in all extremities. Her physical examination is otherwise normal.

Which of the following statements regarding the physical examination findings of fibromyalgia is false?
A. Evidence of synovitis, objective muscle weakness, or other definite physical or neurologic signs suggests the presence of either a comorbid disease or an alternative diagnosis
B. When assessing tender points, palpation is performed with the thumb, using approximately 4 kg of pressure
C. For an accurate diagnosis, the examiner must confirm pain at all 18 tender points
D. Useful tests in fibromyalgia include antinuclear antibody (ANA), complete blood count (CBC), erythrocyte sedimentation rate (ESR), C-reactive protein (CRP), thyroid-stimulating hormone (TSH), creatine kinase (CK), aspartate aminotransferase (AST), and alanine aminotransferase (ALT)

Key Concept/Objective: To know the important components of the physical examination of a patient with fibromyalgia

Evidence of synovitis (e.g., joint effusion, warmth over the joint, pain on joint motion), objective muscle weakness, or other definite physical or neurologic signs suggest the presence of either comorbid disease or an alternative diagnosis. Eighteen specific tender points have been identified in fibromyalgia. A patient with fibromyalgia will have pain, not just tenderness, on palpation at many of these tender points. Palpation is performed with the thumb, using approximately 4 kg of pressure—about the pressure necessary to blanch the examiner’s thumbnail. Attempting to confirm pain at all 18 tender points is not necessary for diagnosis and is inconsiderate toward patients, many of whom find tender-point palpation quite distressing. Useful tests in fibromyalgia include the following: ANA, CBC, ESR, CRP, TSH, CK, AST, and ALT. Tests for Lyme disease, Epstein-Barr virus infection, and endocrinologic status are usually unnecessary. (Answer: C—For an accurate diagnosis, the examiner must confirm pain at all 18 tender points)

For more information, see Winfield JB: 15 Rheumatology: XIII Fibromyalgia. ACP Medicine Online (www.acpmedicine.com). Dale DC, Federman DD, Eds. WebMD Inc., New York, July 2004